You are on page 1of 36

GAZETA DE FISICA

R E V I S T A D O S E S T U D A N T E S D E F S I C A
E DOS F SI COS E TCNI CO- F SI COS PORTUGUESES
VOL . I , F A SC. 8
J U L H O , 1 9 4 8
FOTOGRAFIA SOLARIZADA
(v. pg. 235)

G A Z E T A D E F S I C A
Vol. I, Fasc. 8 Julho de 1948
SUMRI O
1. Tribuna da Fsica
A propsito dum aniversrio por A. G. 225
2. Ensino Mdio da Fsica
cerca do estudo educativo da fsica por A. Silveira Ramos 226
4. Exames do Ensino Mdio
Pontos de exames do curso complementar de cincias
Resolues de Rmulo de Carvalho 229
5. Exames Universitrios
Pontos de exames Resolues de Glaphyra Vieira 229
8. Divulgao e Vulgarizao
Sobre as noes de velocidades de grupo e de fase
por Ldia Salgueiro 231
Solarizao por A. Esteves de Souza. 235
La radioactivit et le remplissage des trous du sys-
tme priodique por M. Hassinsky 238
9. Histria e Antologia
Frederico Paschen por M. H. S. 241
Sobre o objecto, mtodo e estudo da fsica por Harley Howe 242
10. Qumica
A acstica e a qumica por Fernando Neves da Silva 243
Problemas de exames universitrios Resolues de Alice
Maia Magalhes e Chagas Roquete 245
Pontos de exames do curso complementar de cincias
Resolues de Rmulo de Carvalho 246
11. A Fsica nas suas Aplicaes
LAcoustique des salles por P. Mariens 247
12. Informaes Vrias 255
A matria de cada artigo tratada sob a inteira responsabilidade do autor.
RESPONSVEIS DAS SECES
1. TRIBUNA DA FSICA
Armando Gibert
2. ENSINO MDIO DA FSICA
J. Xavier de Brito
3. ENSINO SUPERIOR DA FSICA
F. Soares David, L di a Sal -
gueiro e Antnio da Silveira.
4. EXAMES DO ENSINO MDIO
Rmulo de Carvalho
5. EXAMES UNIVERSITRIOS
Carlos Braga, Joo de Almeida
Santos, Mrio Santos, Jos Sar-
mento e Glaphyra Vieira
6. PROBLEMAS DA INVESTIGAO EM
FSICA
Manuel Valadares
7. PROBLEMAS PROPOSTOS
Amaro Monteiro
8. DIVULGAO E VULGARIZAO
Rmulo de Carvalho
9. HISTRIA E ANTOLOGIA
Francisco Mendes
10. QUMICA
Alice Maia Magalhes, Afonso
Morgenstern e Marieta da Sil-
veira.
11. A FSICA NAS SUAS APLICAES
Carlos Assuno, Ruy Lus Go-
mes, Kurt Jacobsohn, Flvio Re-
zende, Hugo Ribeiro e Manuel
Rocha.
12. INFORMAES VRIAS
Direco
DIRECO: Jaime Xavier de Brito, Rmulo de Carvalho, Armando Gibert e Ldia Salgueiro
TESOUREIRO: Carlos M. Cacho; SECRETRIOS: Carlos Jorge Barral e Maria Augusta Prez Fernndez
COLABORADORES DO ESTRANGEIRO: Miguel Cataln (Madrid), A. Van Itterbeek (Louvain), Jean
Rossel (Zrich), Pierre Demers (Montral Canad), Marcel L. Brailey, (Pittsfield, Mass. U. S. A.)
PROPRIEDADE E EDIO: Gazeta de Matemtica, Lda.
Correspondnci a di ri gi da a GAZETA DE FSICA Lab. de Fsi ca F. C. L. R. da Escol a Pol i tcni ca LISBOA
NMERO AVULSO ESC. 10$00 Assinatura: 4 nmeros (1 ano) Esc. 30$00
Deposi t r i o: LIVRARIA ESCOLAR EDITORA Rua da Escol a Pol i t cni ca, 68-72 Tel . 6 4040 LISBOA

EM TODO O BRASIL: NMERO AVULSO 10 CRUZEIROS
Di st r i bui dor excl usi vo: LIVRARIA BOFFONI B. Car i as & C. Lda. Rua Chi l e, 1 RIO DE JANEIRO
T i p o g r a f i a Ma t e m t i c a , L d a . Ru a Al mi r a n t e Ba r r o s o 2 0 r / c . L i s b o a - N

GAZ E T A DE F S I CA
F u n d a d o r : A R M A N D O G I B E R T
Direco: J. Xavier de Brito Rmulo de Carvalho Armando Gibert Ldia Salgueiro
Vol . I , Fasc. 8 Jul ho de 1948
1 . T RI B UNA DA F S I CA
A P R O P SI TO D U M A N I V E R S R I O
ciaes nacionais de cientistas, visto que a
nova organizao deveria ser uma Federao
de organizaes e no de indivduos.
O Conselho Executivo, nomeado em Julho
de 1946, depressa elaborou uns Estatutos,
hoje j aprovados, e dos quais nos parece de
maior interesse transcrever algumas disposi-
es, relativas ao objectivo (2) e aos mem-
bros (6).
2. A finalidade da Federao a promo-
o da compreenso e cooperao entre as
organizaes membros no sentido de apoia-
rem a Federao e os seus membros consti-
tuintes na realizao das seguintes funes:
a) Trabalhar para a mais completa utiliza-
o da Cincia na promoo da paz e bem-
-estar da Humanidade, e, especialmente, asse-
gurar-se que a Cincia aplicada na soluo
dos problemas urgentes do nosso tempo.
b) Promover a cooperao internacional
na Cincia e na Tecnologia, em particular
mediante ntima colaborao com a Organi-
zao Educativa, Cientfica e Cultural das
Naes Unidas.
c) Encorajar o intercmbio internacional
dos conhecimentos cientficos e dos trabalha-
dores cientficos.
d) Defender e apoiar a liberdade e coor-
denao do trabalho cientfico, tanto do ponto
de vista nacional como internacional.
225
Nos dias 20 e 21 de Julho de 1946, dele-
gados de 18 associaes cientficas, represen-
tando 14 pases, reuniram-se em Londres e
nomearam um Conselho Executivo encarre-
gado de elaborar os estatutos da Federao
Mundial dos Trabalhadores Cientficos.
Parece-nos ser dever, naturalmente incluido
nos objectivos fundamentais da Gazeta, dar
conhecimento aos nossos leitores da existncia
da nova organizao bem como dos seus fins.
A ideia de formar uma tal associao sur-
giu sem dvida no esprito de numerosos
cientistas de vrios pases, pois, para todos,
cada vez se tornava mais evidente a necessi-
dade duma colaborao organizada.
No entanto, estes desejos s foram concre-
tizados em 1946, Fevereiro, quando duma
conferncia, promovida pela Associao Bri-
tnica dos Trabalhadores Cientficos, sobre
o tema geral Cincia e Bem-Estar da Hu-
manidade
Nesta conferncia, em que tomaram parte
delegados de 9 naes, alm das mais repre-
sentativas figuras da Cincia britnica, reve-
lou-se o desejo unnime de criar uma Fede-
rao Mundial dos Trabalhadores Cientficos.
Resolveu-se ento que a Associao Britnica
dos T. C. se encarregasse de organizar um
projecto de constituio bem como de convo-
car oportunamente delegados de vrias asso-

Vol. I, Fasc. 8 GAZETA DE F S I CA Julho, 1948
e) Encorajar aperfeioamentos no ensino
das cincias e difundir a cultura cientfica e
as suas consequncias sociais entre os povos
de todas as naes.
f) Conseguir uma mais ntima ligao en-
tre as cincias naturais e sociais.
g) Melhorar os estatutos profissionais, so-
ciais e econmicos do trabalhador cientfico.
h) Encorajar os trabalhadores cientficos
a tomarem uma parte activa em negcios p-
blicos e torn-los mais conscientes e mais
responsveis relativamente s foras progres-
sivas que actuam na nossa sociedade.
6. As organizaes que pretendam ser
membros da Federao devem requerer a sua
admisso ao Conselho Executivo... A deci-
so do Consellho Executivo deve tomar em
considerao as seguintes regras:
a) Um trabalhador cientfico considerado
como um indivduo convenientemente qualifi-
cado que se dedica a um trabalho profissio-
nal nas cincias naturais ou sociais, puras ou
aplicadas, incluindo o ensino da Cincia.
b) Considera-se indivduo convenientemente
qualificado todo o indivduo que possui um
ttulo universitrio ou outro reconhecido equi-
valente.
c) A questo de saber se, pessoas ocupadas
em campos de trabalhos limtrofes, podem
considerar-se trabalhadores cientficos deve
ser examinada, para cada domnio particular
de trabalho, luz do respectivo estatuto aceite
no paz ao qual pertence a organizao em
considerao. Se esses assuntos tm o patro-
cnio da Academia das Cincias ou de algum
organismo equivalente, no paz em questo,
podem ser normalmente classificados como
assuntos cientficos para os fins desta clusula.
e) Uma organizao que inclui entre os
seus membros pessoas que, embora dedican-
do-se a trabalho cientfico, no sejam inteira-
mente qualificadas, pode ser membro da Fe-
derao mas os seus direitos de voto e a sua
cota sero determinados em funo do nmero
dos seus membros inteiramente qualificados.
Dum folheto, publicado em 1947 pela Fede-
rao, destacamos ainda o seguinte objectivo
futuro:
Carta dos Cientistas definindo os seus
direitos e deveres. Os standards de ins-
truo, remunerao, condies de trabalho
e oportunidades de colocao variam gran-
demente pelo mundo fora. Na ndia, por
exemplo, um graduado em Cincias um
previlegiado se consegue um emprego cient-
fico, e, se assim fr, quse sempre muito
mal pago. Em certos outros pases as con-
dies so melhores mas h muito poucos
em que elas possam considerar-se satisfa-
trias....
So desde j membros da Federao, asso-
ciaes dos seguintes pases: frica do Sul,
Canad, Checoeslovquia, China, Dinamarca,
Estados Unidos da Amrica do Norte, Frana,
Gran-Bretanha, Grcia, Holanda, ndia, Nova
Zelndia e Sucia representando um total de
mais de 110 mil membros individuais.
A. G.
2. ENSINO M DIO DA FSIC A
C ERC A D O ESTUD O ED UC A TI V O D A F SI C A
Mais vale uma cabea forte
do que uma cabea muito cheia
Montaigne
Um fsico francs referindo-se a Jlio Verne
disse estas palavras cheias de bom senso e de
fecundos ensinamentos: foi ele o primeiro que
soube mostrar-me a Cincia divertida e
amvel, ao passo que tantos sbios no a
mostram seno massadora e solene.
226
de toda a convenincia que todos aqueles
que se dedicam a ministrar conhecimentos de
Fsica, ou de qualquer outro ramo do saber
humano, nunca percam de vista a extraordi-
nria importncia do factor psicolgico, pois
que reduzir o estudo apenas aquisio dos

Vol. I, Fasc. 8 GAZETA DE F S I CA Julho, 1948
conhecimentos, sem atender, por um lado, ao
despertar do intersse, e por outro ao val-
or formativo do estudo da Cincia deixar
perder a magnfica oportunidade que se nos
oferece para a Educao da Inteligncia e
para a aquisio de hbitos de observao
e de raciocnio, de indiscutvel utilidade pr-
tica para os indivduos e para as Naes.
Chamo intencionalmente estudo da Fsica
e no ensino, no v esta palavra tradu-
zir-se por preleco que o aluno ouve passiva-
mente, sendo o seu crebro como urna que
se enche de conhecimentos, quando devia ser
antes como oficina de cujo esforo resulte
a aquisio dos conhecimentos que preten-
demos dar.
O professor, dentro desta concepo, um
orientador dos esforos, reduzindo a um mnimo
as afirmaes baseadas na sua autoridade,
para que o aluno consiga um mximo de saber
baseado na sua prpria observao, na sua
prpria experincia e no prprio jogo das suas
faculdades.
O facto de dizer que o professor apenas
um orientador de esforos pode parecer a
quem julgue pelas aparncias, que de certo
modo o professor fica diminuido; mas per-
feitamente o contrrio que sucede, pois
muito mais difcil e exige muito mais talento
pedaggico fazer trabalhar os crebros, ginas-
ticando-os e robustecendo-os pelo exerccio
de modo a aumentar a sua capacidade mental
do que preleccionar sobre dada matria, colo-
cando-os numa atitude meramente passiva.
Se no primeiro caso necessrio bagagem
cientfica e pedaggica, no segundo basta ape-
nas bagagem cientfica, que pode coexistir,
evidentemente, com a ignorncia absoluta da
Pedagogia; por isso tantos sbios s conce-
bem e s mostram a Cincia massadora e
solene....
Expostas as razes porque prefiro o voc-
bulo estudo ao vocbulo ensino, passo a
justificar a razo por que juntei o adjectivo
educativo.
A iniciao ao estudo da Fsica que me
obrigaram a fazer foi muito pouco educativa:
meteram-me numa sala, entre aquelas clssicas
quatro paredes, onde se pode estudar bem
Portugus ou Latim, Histria ou Lnguas,
mas nunca Fsica, e ali despejaram, a Cincia
dentro da minha pobre cabea. Eu no me
interessava pela Fsica! Pudera! No se
pode estudar Fsica a observar as figuras
dos livros, ou a fazer riscos com giz num
quadro preto...
Ver os aparelhos parados j um passo
em frente mas ainda no nada; v-los a
funcionar mais outro, mas no sejamos ing-
nuos supondo que numa aula de 40 alunos
possvel observar convenientemente certas
minuciosidades que so de grande importn-
cia para que se realize um estudo conveniente
da Fsica.
Mesmo que isso fosse possvel, no seria
suficiente estudar Fsica, vendo fazer expe-
rincias ao mestre: seria como aprender a
nadar... vendo nadar os outros.
As lies de Fsica so de vrios tipos den-
tre elas so frequentes quer as que consistem
em descries de aparelhos, quer aquelas em
que se faz o estudo das leis.
Se queremos fazer o estudo da mquina a
vapor, primeiro deve aparecer funcionando,
a mquina a vapor; no h descrio que
possa substituir a viso directa das realidades.
De maneira anloga se dever fazer sempre
que se pretenda realizar o estudo de aparelhos.
Semelhantemente, com o princpio de Ar-
quimedes, por exemplo, devemos comear
pela observao dos factos. Apoiados nos
factos e em face de resultados obtidos, con-
duzido pelo professor, o aluno concluir a lei.
Enunciar o princpio de Arquimedes para
depois fazer a sua verificao experimental,
pode ser eficiente, vistas as coisas sob o n-
gulo informativo. Contudo observadas sob
o ngulo formativo isto uma monstruosi-
dade no se pem em jogo as actividades
mentais do aluno; no se faz chegar, pelo
seu prprio esforo concluso; no se criam
hbitos de raciocnio nem de crtica; um
estudo livresco no confundir com terico
seguido duma experincia cujo resultado j se
sabia e foi adquirido sem actividade psquica.
Duma maneira geral, podemos dizer pri-
227

Vol. I, Fasc. 8 GAZETA DE F S I CA Julho, 1948
meiro os factos este um princpio basilar,
no s porque actuar em primeiro lugar sobre
os sentidos despertar um maior interesse,
mas tambm porque o caminho natural do
esprito humano partir pela observao dos
casos particulares para as leis gerais.
Deixar de seguir este caminho, forar o
esprito a trajectrias artificiais, onde a auto-
ridade imposta pelo mestre vai fatalmente
realizar o encher da urna, fazendo desapa-
recer todo o jogo da capacidade de observa-
o e de experimentao, bem como a reali-
zao do esforo para chegar Verdade,
esforo esse muitssimo fecundo para o desen-
volvimento mental que no s vale pelo menos
tanto como a aquisio do conhecimento,
como tambm fica mais slido, mais seguro
e mais pronto a ser aplicado quando neces-
srio for a assuntos at muitssimo diferentes.
Parece-me que nesta altura j quase se
torna desnecessrio insistir na razo porque
chamei estudo educativo da Fsica e no ape-
nas ensino da Fsica.
O estudo educativo da Fsica s existe
quando se observa, se experimenta, se tiram
concluses primeiro os factos! No im-
porta que as concluses sejam erradas;
mesmo necessrio que se erre; dos erros
tiraremos concluses erradas, como Socrates
no esqueamos nunca a sua maneira de
conduzir as inteligncias na procura da Ver-
dade pois s assim se procurariam com inte-
resse as novas solues, ficando, de todo este
labor mais slidas as aquisies dos conheci-
mentos, e mesmo que no venham a ser
aplicados! nunca trabalho perdido para
desenvolver a capacidade de bem raciocionar.
interessante constatar o vivo interesse
que os alunos tomam pela observao e pela
experimentao: os olhos abrem-se com
vivacidade, nos rostos h alegria, que mostra
que no esprito h prazer, esse prazer intelec-
tual que necessrio cultivar, no para o
transformar num fim, mas para suavizar o
esforo que sempre necessrio para per-
correr os caminhos rduos da Cincia.
Tornar este esforo agradvel eis o que
importa, mas seria mau cair no erro oposto,
228
que dar a impresso que o estudo da Cin-
cia apenas prazer.
Se me pedissem uma comparao, com a
qual concretizasse melhor o meu pensamento
compararia o estudante de Fsica ao alpinista
que realiza esforo mas sente o prazer do
Movimento, do Ar Livre, da grandiosidade
dos panoramas observados do alto e da vit-
ria sobre as dificuldades; e no a um homem
carregado com uma enorme mala a fazer um
imenso frete...
So duas atitudes psicolgicas, completa-
mente diferentes, que importa distinguir.
Tudo, enfim, deve ser aproveitado para
que se tire do estudo da Fsica o mximo do
rendimento educativo desde o cuidado espe-
cial com a primeira lio construo de
aparelhos simples realizados pelos alunos
e desde a interpretao de passagens de livros
de Jlio Verne at Fsica que constante-
mente encontramos na vida corrente, sem es-
quecer a Fsica Recreativa, com os seus enor-
mes atractivos.
Com quarenta alunos nas turmas, sem tra-
balhos prticos ou mesmo com turnos de vinte
alunos nos trabalhos prticos, muito se perde,
mesmo que o professor esteja integrado no
esprito formativo da Fsica; mas nem por
isso deixa de ser vantajoso meditar e focar
os assuntos desta natureza.
Em Portugal, temos entre os nossos defeitos
um que pode e deve ser largamente comba-
tido pelo estudo educativo das Cincias expe-
rimentais refiro-me grande tendncia para
o livresco.
A Fsica presta-se, quando estudada com
inteligncia, a desenvolver em ns o sentido
do real, aumentando a capacidade de realizar
e fomentando a aptido tcnica, com forte
projeco no aproveitamento da riqueza e no
progresso da Nao.
Precisamos, pois, de cultivar o estudo edu-
cativo da Fsica e de lhe dar meios para o
tornar eficiente.
No ser esta uma faceta interessante da
Educao Nacional?
A. C. DA SILVEIRA RAMOS
PROFESSOR DO LICEU DE PASSOS MANUEL

Vol. I, Fasc. 8 GAZETA DE F S I CA Julho, 1948
4 . EX A M ES D O EN SI N O M D I O
PO NTO S DE EXA M ES DO C URSO C O M PLEM ENTA R DE C INC IA S
Liceus de Lisboa Junho de 1948 1. chamada.
53 I) Uma esfera metlica de 5 centmetros de
raio est carregada de electricidade ao potencial de
1200 volts. Considere, no espao, dois pontos A e B
que distem, respectivamente, 10 centmetros e 12 cen-
tmetros do centro da esfera. Calcule: 1.) a intensi-
dade do campo elctrico no ponto A devido carga
da esfera. 2.) a intensidade da fora, expressa em
gramas, com que a carga da esfera actuaria sobre
uma carga pontual de 72 U. Es. Q. colocada no ponto B.
3.) a diferena de potencial, expressa em volts, entre
os pontos A e B. 4.) a quantidade de calor desen-
volvida na descarga total da esfera se toda a energia
elctrica nela contida fosse tranformada em calor.
(J = 4,18 J/cal). Todas as aces elctricas considera-
das se passam no vcuo. R: Carga elctrica da esfera:
Q = CV = 51200:300 = 20 U. Es. Q. 1) E = Q:d
2
=
= 20:10
2
= 0,2 U.Es.E.; 2) F = 2072:122 = 10 dynes =
= 1:98 g; 3) VAVB = 20:1020:12 = 1:3 U.Es.V. =
=100 V; 4) W = QV:2 = 201200:2300 = 40 ergs;
Quantidade de calor: 40:4,1810
7
cal.
54 II) Tema de desenvolvimento: Unidades de
Presso. Desenvolva o assunto de acordo com as se-
guintes alneas: a) que entende por presso; b) defi-
nio e relao entre as unidades de presso nos sis-
temas C. G. S., M. K. S. e M. Kp. S.; c) unidades de
presso empregadas frequentemente na medida das
presses dos vapores nas caldeiras; d) unidades de
presso empregadas na medida da presso atmosfrica.
Deduo da relao numrica entre elas. (Massa es-
pecifica do mercrio =13,6 g/cm
3
).
Liceus de Lisboa Junho de 1948 (2. chamada).
55 I) Pretende-se elevar a gua dum poo por
meio duma bomba. Para isso, liga-se directamente a
bomba ao eixo dum motor elctrico de corrente cont-
nua que trabalha a 100 volts com uma corrente de 5
ampres e cuja resistncia de 4 ohms. Calcule:
1.) A potncia perdida no interior do motor e que
se supe ser apenas devida ao efeito Joule. 2.) Quantos
litros de gua elevar a bomba ao fim de um quarto
de hora de funcionamento, sabendo que de 3 metros
a altura a que se pretende elev-la e sabendo tambm
que o motor consome 20 watts para vencer as vrias
resistncias da instalao. R: 1) Potncia perdida =
= i
2
r = 5
2
4 = 100 W; 2) Potncia, total = Vi = 500 W;
Potncia til: 50010020 = 380 W; Trabalho reali-
zado em 1/4 de hora: 3801560 = 34210
3
joules;
massa de gua elevada: de W = mge vem m = W:ge =
= 34210
3
:9,83 = 11410
2
kg que equivalem a 114
10
2
litros de gua.
56 II) Tema de desenvolvimento: Trabalho efec-
tuado pelos binrios. Desenvolva o assunto de acordo
com as seguintes alnea: a) Definio de binrio, de
brao dum binrio e de momento dum binrio;
b) Como se calcula o trabalho realizado por um bin-
rio; c) Em que unidades devemos indicar os valores
das grandezas que entram nesse clculo para que o
trabalho venha expresso em ergs, em joules ou em
quilogrmetros.
Resolues de RMULO DE CARVALHO
5 . EXA M ES UN IV ERSIT RIO S
P O N TO S D E E X A M E S
F. C. L. Fsica F. Q. N. Exame final Junho 1948
152 a) Defina equilbrio radioactivo e diga
o que entende por equilbrio secular e transitrio.
b) Descreva o acelerador circular; explique o seu fun-
cionamento. c) Indique a partir da definio de frac-
o de sntese se a reaco de decomposio dum ele-
mento nas suas partculas contituintes uma reaco
exotrmica ou endotrmica. Justifique a sua resposta.
153 a) Explique como se determina o nmero de
Avogadro pelo mtodo dos raios X; b) Diga como
varia o nmero de massa e o nmero atmico dum
elemento que se transmuta por emisso e justifique
a sua resposta.
154 Aniquilao da matria e materializao da
energia.
229

Vol. I, Fasc. 8 GAZETA DE F S I CA Julho, 1948
F. C. L. Mecnica Fsica Exame Final Julho
1948.
155 De cada um dos extremos duma corda que
passa por uma roldana esto suspensos 80 kg. Que
sobrecarga ter que se aplicar para que a rol-
dana gire? O raio da roldana 10 cm e o do eixo
1,0 cm. O coeficiente de atrito da roldana com o eixo
0,105. R: Quando se aplica a sobrecarga mnima
P para que a roldana comece a rodar, o ponto de apli-
cao da resultante passa de O para O e no ponto de
contacto do eixo com a roldana a resultante pode decom-
pr-se na normal N e na fora de atrito A .
| |
( ) | |
| | ( ) ( ) 0 ; 0 rot
rot grad
2
1
grad
const
1 1
2
; rot grad
2
1
2
2
2
= V V' +
c
c
= +
c
c
)
`

+
c
c
= +
= + + =
V b b V
t
b
V b
t
b
V V v
t
V
p V
p v
v
V V v


( ) 0 = V V b
dt
b d

159 Ondas acsticas planas; frmulas de Laplace.
Expresses a utilizar:
( ) ; ;
2
2
0
x ct
t
=
c
c
= A


. ; const ;
1
0
M qRT qp c pv c
q
= = = =


F. C. L. Termodinmica Exame final Julho 1948
160 Um sistema mantido a presso constante re-
cebe uma quantidade de calor Q12. Demonstrar que
o aumento de entropia H2H1= Q12.
161 Quanto vale a variao de entropia espec-
fica numa corrente fluida adiabtica quando no haja
produo de trabalho exterior nem variaes de nvel
e em que as velocidades de entrada e de saida so
iguais.
162 Defina-se temperatura de saturao adiab-
tica e deduza-se a relao
e sa
q
r
r r
45 , 0 24 , 0 +
'
=
su-
postas conhecidas as expresses:
( ) ( ) ( )
kcal/kg 24 , 0 kcal/kg; 45 , 0 595
1 1 1
+ = + =
= ' = ' + '
a v
e v v v v a a a
h h
q m h h m h h m h h m

163 Aplique a regra das fases s dissolues e
defina o que se entende por pontos de eutexia.
164 Mediante as expresses:
. 0
; 0
1 1
2
2
1 1 1
1
= ' '
' '
' '
' ' '
'
'
' + A +
= ' ' ' ' ' ' + ' ' ' A +
c d
c
M
c d
c
M
dp v dT
T
Q
c d M c d M dp v dT
T
Q



Explique a marcha da destilao duma mistura de
dois lquidos quando se opere a presso constante.
F. C. L. Electricidade Exame final Julho, 1947
165 a) Estabelea as equaes de transformao
da acelerao. b) Aberrao das estrlas. c) Massas
dum ponto material.
166 a) Diga como varia com a temperatura o
nmero de electres emitidos dum filamento b) Teoria
do Efeito Compton. c) Lei de Moseley; energia to-
tal dum electro no campo do ncleo.

O coeficiente de atrito entre o eixo e a roldana dado
por f = A/N = tag sendo o ngulo de atrito entre o eixo
e a roldana. Tem-se ento: P+P/P = (r1+x)/(r1x)
sendo x = r2 sen . Substituindo x pelo seu valor e con-
siderando tag = sen por se tratar de ngulos muito
pequenos vem: (P+P)/P = 1,02 donde P = 81,6
80,0 = 1,6 kg.
156 Forma da superfcie lquida na proximidade
de uma parede plana.
Expresses a utilizar:
( ). cos 1
2
1
; sen
2

= = = gz
dz
d
r
gz
157 Equaes fundamentais da hidrodinmica.
Teorema de Bernoulli.
Expresses a utilizar:
| | V V v p F rot grad
2
1
grad
1
2
=


3 2 1 1
3
3
2
2
1
1 1
1
x
v
v
x
v
v
x
v
v
x
p
F
c
c
+
c
c
+
c
c
=
c
c


..
..
158 Conservao da massa dos remoinhos:
Expresses a utilizar:
= = p V V F grad
1
grad
1
; grad
1


230

Vol. I, Fasc. 8 GAZETA DE F S I CA Julho, 1948
167 a) Diga como se varia o comprimento de
onda prprio de uma antena. b) Origem do espectro
contnuo de raios X. c) Raios csmicos.
168 Sendo v=0,8c determine em S a equao
da curva x
2
+y
2
=251 e calcule a rea que limita na-
quele referencial. R: Em S a equao x
2
+y
2
=25
representa uma circunferncia de raio igual a 5.
No sistema S tem-se y = y e x = xR sendo R =
.

Substituindo valores e efectuando operaes vem para
mente. A rea desta curva em S ab = 3,14
35 = 47,1.
169 Calcule a energia que devia emitir durante
1 segundo um electro que com movimento uniforme
percorresse uma rbita circular de 1,00 AU em 10
10

segundos: e = 1,59210
20
U. Em. R: A energia
emitida por um electro em movimento durante o tempo
e com a acelerao j dada por: w=2e
2
j
2
/3v (1).
A velocidade com que o electro percorre a rbita
dada por v = 2r/t = 210
8
/10
10
= 210
2
cm/s e a
acelerao nestas condies j = v
2
/r = 4
2
10
12
cm/s
2
.
Substituindo valores em (1) vem para a energia emitida
num segundo: W = 41810
18
ergs.
Resolues de GLAPHYRA VIEIRA
c v 1
2 2


(1) em S a equao:

1
5
y
3
x
2
2
2
2


A curva uma elipse de semi-eixos 5 e 3 respectiva-
8. DIVULGAO E VULGARIZAO
SO BRE A S NO ES DE VELO C IDA DES DE G RUPO E DE FA SE
B
1
, B
2
fig. 1 a). evidente que nesse mes-
mo instante, os outros mximos (C
1
e C
2
, etc.)
da varivel luminosa correspondentes a cada
uma das radiaes, no coincidem, visto que
os comprimentos de onda das duas radiaes
so diferentes.
A perturbao resultante das duas compo-
Para uma radiao rigorosamente monocro-
mtica, a relao entre o comprimento de onda
, a velocidade v de propagao da onda e o
perodo T, , como se sabe,
=vT.
Imaginemos que a origem emite simultnea-

nentes est representada na fig. 1 b. V-se
imediatamente da figura, que enquanto que
as perturbaes componentes correspondem
mente duas radiaes de comprimentos de
onda
1
e
2
, pouco diferentes.
Consideremos um ponto do espao, em que

num dado instante t, as variveis luminosas
correspondentes s duas radiaes tenham
simultneamente os valores mximos (ponto
a ondas com amplitude constante, a pertur-
bao resultante constituida por uma onda
cuja amplitude passa por mximos e mnimos.
231

Vol. I, Fasc. 8 GAZETA DE F S I CA Julho, 1948
radiao de comprimento de onda
1
a sua
velocidade de propagao que designaremos
por V diferente da velocidade v
1
; e como
este raciocnio se poderia reproduzir conside-
rando em primeiro lugar a radiao de com-
primento de

onda
2
, fcil de ver que
tambm V v
2
.
Na fig. 2 esto representados: em a, os
pontos B
1
, B
2
da fig. 1 a e as posies B
1
, B
2

destes pontos no instante t + (B
1
B
1
= v
1
;
B
2
B
2
= v
2
); em b a onda resultante no ins-
tante t + (BA = V), sendo A a posio
Consideremos agora dois casos:
a) O das radiaes se propagarem no
vcuo;
b) O das radiaes se propagarem num meio
material.
a) No primeiro caso, as duas radiaes pro-
pagam-se com a mesma velocidade c, e, por
consequncia, aps um intervalo de tempo
qualquer , (instante t + ) os dois mximos
das duas curvas que coincidiam, estaro ambos
a uma distncia c, isto , continuaro ambos
em sobreposio, e ser a adio destes dois

mximos que corresponder ainda ao valor do
mximo de amplitude da onda resultante.
velocidade com que se desloca este m-
ximo chama-se velocidade do grupo para as
duas radiaes consideradas, que como vemos
a velocidade c com que se propagam as
ondas.
b) Designemos por v
1
e v
2
as velocidades
de propagao das radiaes de comprimentos
de onda
1
e
2
. Consideremos de novo a
direco de propagao; no fim de um certo
tempo , o mximo da radiao de compri-
mento de onda
1
estar distncia v
1
, mas
o mximo da radiao
2
j agora no estar
sobreposto ao anterior, visto ter percorrido
a distncia diferente v
2
. Daqui resulta que
o mximo da onda resultante j no se encon-
tra em B
1
, B
2
como na fig. 1 a, estando antes
ou depois desse ponto. Por consequncia,
como o mximo da onda resultante percorreu
uma distncia diferente da percorrida pela
232
do mximo da onda resultante no referido
instante.
Em resumo, no vcuo, as velocidades das
ondas e a velocidade do mximo da onda
resultante so iguais, ao passo que num meio
material essas velocidades so diferentes.
O problema que se pe o de determinar
a relao que existe entre V, v
1
e v
2
; mas
antes de passarmos a este clculo, analise-
mos mais de perto o significado fsico do que
acaba de ser exposto, dando algumas imagens
mecnicas.
Consideremos dois veculos que se movem
independentemente e suponhamos que existem
trs referncias, uma ligada ao veculo A
(fig. 3) outra ao veculo B e finalmente uma
terceira que est permanentemente a meia dis-
tncia dos dois veculos. Suponhamos ainda,
para libertar o esprito de qualquer influncia
relativa posio dos veculos, que o obser-
vador se encontra numa posio tal que no

Vol. I, Fasc. 8 GAZETA DE F S I CA Julho, 1948
v os veculos, mas exclusivamente as trs
referencias.
No primeiro caso (deslocamento no vcuo)
os dois veculos movem-se com a mesma velo-
cidade, e, por consequncia, passado um certo
intervalo de tempo as trs referncias encon-
tram-se mesma distncia das posies iniciais
(fig. 3a).
No segundo e terceiro casos (deslocamento
da onda resultante, supondo porm que a forma
da onda se modifica medida que o veculo
se move. Para estabelecermos a analogia com
a mecnica, imaginemos agora o caso de um
nico veculo, mas em que a referncia em
vez de ocupar uma posio fixa no veculo,
varia durante o movimento deste.
Convm que o leitor nunca se esquea de
que os veculos so para ele invisveis, s

num meio dispersivo) os dois veculos marcham
com velocidades diferentes, e, por consequn-
cia as distncias percorridas no mesmo inter-
valo de tempo pelas trs referncias sero
tambm diferentes (fig. 3b). O cociente da
distncia percorrida pela referncia de A pelo
tempo, dar-nos- a velocidade do veculo A.
Anlogamente para B e, finalmente, o cociente
da distncia percorrida pela referncia 2 pelo
mesmo intervalo de tempo, dar-nos- a velo-
cidade de grupo. A figura mostra que a
velocidade V estar compreendida entre v
1

e v
2
, sendo, no caso b, v
1
< V < v
2
e no caso c,
v
1
> V > v
2
.
O problema pode ser visto ainda segundo
um outro aspecto; afinal, o facto da veloci-
dade do grupo ser diferente da velocidade
das ondas, corresponde onda resultante se
deformar durante o percurso. Se abstrairmos
pois das duas perturbaes componentes,
poderemos considerar smente o movimento
podendo avaliar das velocidades pela posio
das referncias.
A figura 4 corresponde precisamente ao
caso, da posio do sinal variar em relao

ao veculo, mas para maior aproximao da
imagem da onda, adoptou-se como sinal o
ponto mximo da cobertura de um veculo,
cobertura esta que se deforma durante a
marcha.
Alm destes exemplos mecnicos, pode
apontar-se um exemplo fornecido pelo reino
233

Vol. I, Fasc. 8 GAZETA DE F S I CA Julho, 1948
No intervalo de tempo t
2
t
1
= t, a onda 2
percorreu a distncia x
2
x
1
+
1
= x +
1
e a
onda 2 percorreu a distncia x +
2
. Final-
mente a onda resultante que no instante t
1

tinha o seu mximo em x
1
, tem no instante
t
2
o seu mximo em x
2
, isto , no intervalo
de tempo t
2
t
1
, percorreu a distncia x.
As trs velocidades so dadas, por conse-
quncia, respectivamente pelas frmulas:
,
, ,
2
2
1
1
t
x
V
t
x
v
t
x
v



donde
.
2 1
2 1
1 1


v v
v V
Se a funo v = f () admitir derivada, supo-
animal: o movimento de uma lagarta
(1)
. Com
efeito, medida que esta avana, a parte
mais elevada do seu corpo vai, em relao a
qualquer das extremidades, variando de posi-
o e, assim, se medirmos a velocidade com
que a lagarta se desloca, tomando como ponto
de referncia uma das suas extremidades,
obteremos uma grandeza que poderemos cha-
mar a sua velocidade de propagao, enquanto
que se o ponto de referncia fr em qualquer
instante o ponto mais elevado do corpo da
lagarta, determinar-se- uma velocidade (velo-
cidade de grupo) que poder ser, conforme os
casos, maior ou menor do que aquela. fcil
de ver, alis, que h uma estreita semelhana
entre o deslocamento da lagarta e o movi-
mento que considermos do veculo, cujo teja-

dilho se deformava medida que aquele
avanava.
Passemos agora deduo da expresso que
relaciona V com v
1
e v
2
.
Consideremos duas radiaes monocrom-
ticas de comprimentos de onda
1
e
2
(
2
>
1
)
que se propagam segundo a direco do eixo
dos xx (fig
.
. 5) Consideremos um instante t
1

em que duas ondas (2 e 2) das duas radiaes
esto em fase no ponto x
1
. Admitamos que
o meio apresenta uma disperso normal, isto
que v
2
>v
1
.
Num instante posterior, como as duas radia-
es marcham com velocidades diferentes,
B
1
e B
2
deixam de estar coincidentes; ao
mesmo tempo, 1 tende a aproximar-se de 1
at chegar o instante t
2
em que 1 e 1 se
sobrepem; seja x
2
o ponto em que tal
acontece.

1)
V. bibliografia.
234
remos que a expresso anterior ainda vlida
no limite, poderemos ento escrever:
.
1 1

d
dv
v V
Designaremos por V a velocidade de pro-
pagao do grupo de ondas, ou velocidade
de propagao de amplitude; v
1
e v
2
so as
velocidades de propagao das ondas monocro-
mticas ou velocidades de propagao de fase.
BIBLIOGRAFIA
1. Compton e Allison, X Rays in Theory
and Experiment.
2. Herbert Jehle, American Journal of
Physics, pg. 47, 1946.
3. G. Bruhat, Optique (1942).
4. R. Wood, Physical Optics (1928).
LIDIA SALGUEIRO
1. ASSIST. FSICA DA F. C. L.

Vol. I, Fasc. 8 GAZETA DE F S I CA Julho, 1948
S O L A R I Z A O
um determinado tempo de exposio, tendo-se
verificado experimentalmente que so dife-
rentes os resultados quando se faz a exposio
a uma nica origem de radiaes monocrom-
ticas ou a mais de uma origem de radiaes
diferentes, sucessivamente. So tambem dife-
rentes os efeitos conforme a solarizao
produzida pela exposio prolongada ou por
radiaes de grande intensidade.
Exemplificando: Se, numa mquina foto-
grfica, exposermos uma placa fotogrfica,
luz natural, por um tempo de 100 a 500 ve-
zes o normal, para a sensibilidade e tipo de
emulso usada, provocamos um enegrecimento
expontneo da emulso, mas, branqueando a
placa com um oxidante enrgico (como por
exemplo o permanganato de potssio) redu-
zimos o negativo nas partes mais enegrecidas
sem que isso afecte a imagem que j est
formada e que de simples imagem latente
negativa passou, pela exposio prolongada,
a imagem positiva como se pode verificar re-
velando agora a chapa com um revelador
vulgar cuja funo nica reduzir o haloge-
neto do prata que no foi transformado em
prata metlica, como em qualquer caso cor-
rente de fotografia.
Um outro caminho para obter resultado
anlogo ao do caso precedente consiste em
comear por expor a placa fotogrfica fora
da mquina a uma fonte de luz branca, forte
e uniforme. Se agora exposermos a placa
assim tratada, sob um diapositivo, por tempo
conveniente, s radiaes infra-vermelhas
(monocromticas ou com muito pouco de ver-
melho visvel), revelando-a em seguida por
qualquer processo corrente, obteremos uma
imagem positiva por solarizao. E num
processo anlogo a ste, em que a luz branca
substituda por tratamento qumico adequado
(chemical flashing) durante a manufactura, que
se funda uma das aplicaes prticas do fen-
meno, de carcter industrial o fabrico das
pelculas chamadas de reverso directa (di-
rect positive films).
235
Ao estudarmos a aco fotogrfica das ra-
diaes depara-se-nos um curioso fenmeno
cujas causas esto ainda mal estudadas cien-
tificamente. Referimo-nos Solarizao,
fenmeno de h muito conhecido dos fsicos e
dos fotgrafos mas que estes sempre conside-
raram como um grave acidente que levava
inutilizao dos seus trabalhos, sem qual-
quer mrito que no fosse a perda pura e
simples do material utilizado.
De alguns anos a esta parte esse acidente
tcnico, que nos meios fotogrficos tambm
conhecido por efeito Sabatier, tem sido
aproveitado por grandes artistas da fotografia,
tais como Max Thoreck, William Mortensen
e Man Ray para s referir alguns dos maio-
res para produzir determinados efeitos ar-
tsticos e originais em alguns dos seus traba-
lhos de Salo.
Por solarizao entendia-se originalmente
uma diminuio do enegrecimento da placa
fotogrfica quando se aumentava a dose duma
certa radiao alm dum certo valor, isto ,
dando uma exposio contnua e to prolon-
gada que a chapa fotogrfica comeasse a
enegrecer expontneamente, a imagem latente
era destruda e se ento revelssemos a
placa fotogrfica, em presena de oxidantes,
aparecer-nos-ia uma estranha reverso, posi-
tiva, total ou qusi total da imagem fotogra-
fada.
Hoje, porm, o vocbulo tem significado
mais lato e emprega-se, indistintamente, tanto
para traduzir o significado clssico como para
os casos de reverso parcial da imagem dos
quais resulta uma curiosa mistura, na mesma
imagem, de positivo e negativo. E ste l-
timo aspecto que tem sido ultimamente explo-
rado sob o ponto de vista artstico.
O fenmeno pode ento definir-se como
sendo uma inverso parcial do fenmeno fo-
togrfico provocado na emulso ou por uma
exposio excessivamente longa a uma dada
intensidade luminosa, ou pela exposio a uma
origem luminosa demasiadamente intensa para

Vol. I, Fasc. 8 GAZETA DE F S I CA Julho, 1948
Deixando de lado as manifestaes do fen-
meno nos casos das radiaes I.V., U.V.,
raios X, e outras monocromticas, que inte-
ressam principalmente aos fsicos, vejamos
como se pode produzir o efeito Sabatier com
radiaes heterocromticas, do domnio do vi-
svel, e seu contrle experimental para fins
puramente fotogrficos (ou de estudo).
O fotgrafo, amador ou profissional, pode
conseguir o efeito, ou solarizando primeira-
mente o negativo e fazendo depois a prova
positiva, ou fazendo primeiramente um nega-
tivo normal e solarizando depois a prova final
mas como, sob o ponto de vista artstico, os
resultados so distintos, o experimentador
fica com a possibilidade de duas escolhas,
consoante o efeito que pretende obter e ainda,
dentro de cada um dos casos pode variar a
intensidade do fenmeno obtendo gradaes
sucessivas do efeito que pretende.
Consideremos:
1) Solarizao do negativo.
De comeo procede-se como para a obten-
o dum negativo normal, expondo a placa
fotogrfica o tempo correcto e depois passan-
do-a para um revelador vulgar mas; daqui
em diante, afastamo-nos do procedimento cor-
rente.
A placa permanece no revelador apenas de
1
/
3
a
2
/
3
, do tempo normal que deve durar a
revelao, que ento se interrompe enxa-
guando a placa em gua limpa, rpida mas
eficazmente, retirando depois o excesso do l-
quido com uma esponja
,
fina ou com uma ca-
mura limpa, para no deixar bolhas, que
seriam outras tantas marcas.
Seguidamente, segurando a placa por um
canto de modo a evitar quaisquer superfcies
reflectoras por detrs, expe-se novamente a
placa, durante um curto perodo, a uma luz que
pode ser a vermelha ou alaranjada da cmara
escura se a emulso fr pancromtica, ou
branca normal se fr ortocromtica. Deve
tomar-se cuidado com a luz reflectida at pela
prpria mo que segura a placa e por isso se
recomenda segur-la apenas por um canto e
236
afastada do corpo ou de qualquer superfcie
reflectora. Depois desta exposio luz de-
volve-se a placa ao banho revelador e con-
clui-se a revelao interrompida. Fixa-se e
lava-se como corrente.
Todo este processo ainda um tanto em-
prico e os resultados dependem de muitos e
variados factores, mas como ponto de partida
podemos indicar :
a) interromper a revelao quando decor-
rido
1
/
2
do tempo normal;
b) sendo a emulso do tipo pancromtico,
de velocidade (sensibilidade) mdia, expor, de
1 a 2 segundos, luz branca duma lmpada
normal de 25 W colocada a cerca de 2 me-
tros ou luz alaranjada duma lmpada de
cmara escura durante cerca de 20 segundos.
Os resultados (gravura da capa) tambem se
podem fazer variar um tanto se, entre a se-
gunda exposio da placa e a concluso da
revelao deixarmos passar desde alguns se-
gundos a cerca de 3 minutos.
Quanto s emulses a empregar so prefe-
riveis aquelas que do maiores contrastes, de
sensibilidade mdia ou fraca, tanto pan
como ortocromticas ou ordinrias.
2) Solarizao das provas positivas.
Faz-se a prova por contacto ou por am-
pliao partindo dum negativo normal, com
bom contraste, maneira ortodoxa utilizando,
de preferncia, papel fino (simples), do tipo
contraste ou duro, e dando-se-lhe a ex-
posio normal adquada.
Num revelador corrente revela-se a prova
at se notarem quse completamente revela-
das as partes mais escuras e nessa altura
retira-se a prova do banho revelador, escor-
re-se, sem lavar, limpando-a apenas do ex-
cesso de revelador com um pedao de borra-
cha macia. Seguidamente expe-se a uma luz
branca por exemplo, luz duma lmpada
de 25 W, colocada a cerca de 1 metro, du-
rante um ou dois segundos e espera-se um
pouco at que as partes claras ou brancas
da prova comecem a escurecer, aps o que
se lhe d uma revelao complementar, mais

Vol. I, Fasc. 8 GAZETA DE F S I CA Julho, 1948
curta que a primeira, num revelador diludo
ou enfraquecido. Fixa-se e lava-se do modo
usual.
O resultado que se obtm ser semelhante
ao da imagem c.

A diferena fundamental entre as provas
que so solarizadas e aquelas que foram obti-
das depois da solarizao prvia do negativo
flores claras, faces, corpos ns devem ser
iluminados com luz difusa colocada qusi por
detrs da mquina). A distribuio da luz
muito importante, pois deve ter-se presente
que a inverso no afecta as partes bem
iluminadas do objecto fotografado mas torna
os brancos, da prova vulgar, em negros, na
prova solarizada.
Finalmento um facto a atender na escolha
do processo a empregar: Solarizando nega-

a b c
A solarizao da prova positiva (c) foi obtida pelo processo descrito em 2) num prova ampliada dum
negativo normal, 69 cm
2
, e a prova do negativo solarizado (capa e b) foi obtida do mesmo negativo
atravs dum negativo intermdio em papel (cujo mtodo de obteno aqui se no descreve) que por
sua vez foi solarizado pelo processo descrito em 1). Para que as solarizaes do negativo, feitas por este
processo, no resultem imagens invertidas como num espelho, da prova directa (a), torna-se necessrio
fazer as provas a solarizar com o negativo colocado ao contrrio,
isto , com a base contra a emulso do papel.
tivos, fcil numa sesso inutilizar uma caixa
de chapas que so incomparvelmente mais
caras do que as flhas de papel e ficamos sem
fotografia; Solarizando provas podemos,
quando muito, perder umas flhas de papel,
mais baratas do que as chapas e uma sesso
de trabalho, mas o negativo, sendo bom, l
est a permitir tirar provas normais, quando
aquelas no dem resultado, isto , mais
fcil obter resultados aproveitveis pelo se-
gundo processo.
A. ESTEVES DE SOUZA
CAND. LIC. CIENC. BIOLGICAS
237
que, embora ambas sejam positivas, o fen-
meno aparece na primeira tal como no nega-
tivo que deu origem segunda, isto , quanto
solarizao, uma negativa em relao
outra, como se pode verificar comparando as
imagens solarizadas b e c.

A quem desejar experimentar este curioso
fenmeno de aconselhar comear com foto-
grafias de assuntos bem contrastados como
os de objectos claros em fundos escuros de-
vendo a iluminao ser simples (por exemplo

Vol. I, Fasc. 8 GAZETA DE F S I CA Julho, 1948
LA RADIOAC TIVIT ET LE REMPLISSAG E DES TRO US DU SYSTME PRIO DIQ UE
Dans lAntiquit on connaissait 8 ou 9 corps
que nous dfinissons actuellement comme l-
ments chimiques. Ce sont le fer (Fe), le
cuivre (Cu), largent (Ag), lor (Au), le plomb
(Pb), ltain (Sn), le mercure (Hg), le soufre
(S), et le carbone (C).
Les 6 premiers et le soufre sont mention-
ns dans la bible. Lobscurantisme du moyen
ge a paralys tout progrs scientifique, et
malgr les nombreuses recherches des alchi-
mistes sur 1a pierre philosophale, ceux-ci
nont ajout la liste; connue que larsenic
(As), lantimoine (Sb), et le bismuth (Bi). Le
phosphore (P) a t dcouvert au XVII
me
sicle.
Les mtaux zinc (Zn), cobalt (Co), nickel (Ni),
manganse (Mn), tungstne (W), chrome (Cr),
molybdne (Mo), uranium (U), tellure (Te),
et les gaz hidrogne (H), oxigne (O), azote (N),
on t isols au sicle suivant.
Lorsque Mendljeff tablit en 1869 le sys-
tme priodique, son tableau contenait 65 l-
ments, compris, entre lhydrogne, llment le
plus lger, et luranium, llment le plus lourd.
Le trait gnial de cette dcouverte consis-
tait non pas dans la classification priodique
des lments daprs leurs proprits les plus
importantes, mais dans les conclusions har-
dies quil en tirait, notamment dans la pr-
vision de lexistence et des proprits de
nouveaux lments, inconnus et correspondant
aux trous quil a laisss dans le tableau
afin de respecter la priodicit. On sait que
Mendljeff avait prvu, entre autres, en d-
tail, les proprits de 3 lments alors incon-
nus: ka-bore, ka-aluminium et ka-sillicium.
Ces lments ont t dcouverts peu de temps
aprs et leurs proprits correspondent exac-
tement celles qui ont t prvues. Ce sont
le scandium (Se), le gallium (Ga) et le ger-
manium (Ge).
Le succs du systme de Mendljeff sest
de nouveau raffermi la fin du dernier si-
cle et au dbut du vingtime avec la d-
couverte de la radioactivit par Becquerel et
Pierre et Marie Curie et des gaz rares, tout
238
une srie de nouveaux lments radioactifs;
le polonium (Po), le radium (Ra), lactinium
(Ac), le protactinium (Pa), et le radon (Rn),
ont pu tre placs sans difficult dans les ca-
ses vides entre le bismuth et le thorium. En
particulier, le gaz radon est venu sajouter
en bas de la nouvelle colonne introduite dans
le systme priodique en 1894-1898 par Ram-
say aprs sa dcouverte des 5 gaz rares de
latmosphre: lhlium (He), le non (Ne),
largon (A), le krypton (Kr), et le xnon (X).
Les lments de cette colonne sont compris
dans le groupe dit zro, ce qui
,
correspond
au point de vue chimique au fait que la valence
de ces lments est nulle.
Le tableau conu par Mendljeff prsen-
tait cependant quelques irrgularits; pour
respecter la priodicit des proprits on
avait t oblig dans 3 ou 4 cas de placer cer-
tains lments avant dautres lments ayant
un poids atomique lgrement plus petit, et
surtout il a t ncessaire de grouper une
dizaine dlments, ayant il est vrai des pro-
prits chimiques trs voisines, dans une et
mme case le groupe des terres rares. Ces
difficults ont t limines lorsque le jeune
savant Moseley (tu aux Dardanelles en 1915)
a montr en 1913 par ltude des rayons X
que les lments chimiques sont bien mieux
caractriss par un numro atomique que par
leur masse atomique. Ce numro correspond
au nombre des lectrons qui gravitent dans
latome autour du noyau central et il peut
tre dtermin par ltude des rayons X.
On a pu ainsi prciser que lhydrogne
correspond le numro atomique 1 et lura-
nium, llment le plus lourd, le numro ato-
mique 92 Entre ces deux lments extr-
mes se classaient tous les autres lments
connus ce moment, soit 84 lments et, en
tout, 86.
Le systme priodique contenait donc alors
6 trous correspondant aux nn 43, 61, 72,
75, 85 et 87. De nombreux chimistes et phy-
siciens se sont mis chercher les 6 lments

Vol. I, Fasc. 8 GAZETA DE F S I CA Julho, 1948
manquants soit dans les minrais soit dans
les produits radioactifs.
Llment 72 a t trouv en 1923 par
Hevesy et Coster Copenhague. Ces deux
savants, grce une comprhension profonde
de la structure atomique, telle quelle rsulte
de la clbre thorie de Niels Bohr, avaient
prvu que ce nouvel lment devait avoir des
proprits chimiques trs proches de celles du
zirconium (Zr). Cest donc dans les minrais
de cet lment quils ont cherch et dcou-
vert llment 72, auquel ils ont donn le nom
de hafnium (Hf) (nom latin de Copenhague).
Son existence a t rvle dabord par l-
tude aux rayons X (prsence de raies pr-
vues par la loi de Moseley), puis abondam-
ment confirme par des mthodes chimiques.
Deux ans plus tard, les poux Noddack
ont dcouvert Berlin, aprs de longues et
patientes recherches, llment 75 quils ont
appel rhnium (Re). Celui-ci a t galement
identifi dabord par les rayons X, puis isol
par des mthodes chimiques. Bien quil soit
un lment assez rare, on pouvait dj avant
la guerre se procurer facilement des quanti-
ts importantes du mtal ou de ses sels. Ces
proprits sont maintenant bien connues: ho-
mologue
(1)
du manganse, il sapproche de lui
par un certain nombre de proprits, mais il
en diffre par bien dautres (la valence VII de
(Re) est beaucoup plus stable, de sorte que
les sels de lacide perrhnique ne sont pas
des oxydants forts comme les permanganates;
il peut former des anions Re

, mais on ne
connait pas dions Mn

).
Ces mmes autours avaient annonc gale-
ment lexistence de llment 43 quils ont
nomm masurium. Mais cette dcouverte base
seulement sur ltude par rayons X na pas
t confirme, et jusquici on na aucune indi-
cation srieuse sur lexistence de cet lment
dans la nature. La mthode des rayons X est,
en effet, dlicate; trs prcieuse pour orienter
la recherche dun nouvel lment, les rsultats

(1)
Les lments qui se trouvent dans la mme
colonne verticale du systme priodique sappellent
homologues.
quelle fournit ne peuvent cependant tre con-
sidrs comme srs que sils sont confirms
par des mthodes chimiques. Cest ainsi que
divers auteurs ont prtendu avoir dcouvert
par rayons X les lments 61 et 87 auxquels
ils se sont dpchs de donner des noms en
honneur de leurs pays ou villes dorigine, mais
dans ces cas galement les faits chimiques nont
pas confirm les dcouvertes annonces. Cer-
tains thoriciens pensent mme que les l-
ments 43 et 61 ne peuvent plus exister dans
la nature par suite de linstabilit de leurs
noyaux atomiques, de sorte que si mme ils
taient prsents dans le globe terrestre lori-
gine de sa formation, ils se seraient depuis
dsintgrs comme corps radioactifs ( vie
relativement courte) et auraient disparu.
Par contre, lexistence de llment 87 a t
prouve avec certitude par Marguerite Perey
en 1939 Paris. Par des mthodes chimiques
et radioactives elle a montr que cest un l-
ment radioactif qui se forme partir de lacti-
nium (Ac), lorsque celui-ci, radioactif lui-mme,
se dcompose par mission dune particule
(qui est un atome dhlium):
(1) 4
2
227
87
227
89
. He Fr Ac
Cet lment sappelle francium (Fr). Comme
il fallait sattendre daprs sa position dans le
systme priodique, il prsente toutes les pro-
prits caractristiques des lments alcalins.
Daprs des expriences toutes rcentes de
Karlik et Bernert faites Vienne, des isoto-
pes radioactifs
(2)
de llment 85 existeraient
galement parmi les produits de dsintgra-
lion du radium, du thorium et de lactinium.
Ce seraient des corps trs instables, dont la
vie moyenne serait de quelques secondes ou
fraction de seconde, de sorte que leur tude
par des mthodes chimiques serait trs diffi-
cile, sinon impossible. Par ce fait mme les

(1)
Le nombre plac gauche et en haut du sym-
bole chimique indique la masse de latome, celui den
bas son nombre atomique.
(2)
Les isotopes sont des varits dun lment qui
diffrent lgrement par leurs masses atomiques; ils
ont pratiquement les mmes proprits chimiques.
239

Vol. I, Fasc. 8 GAZETA DE F S I CA Julho, 1948
expriences en question demandent encore
dautres vrifications.
On a pu cependant prparer cet lment,
qui est un ka-iode au moyen de la radioacti-
vit artificielle et mme tudier ses propri-
ts. Ce sont Corson, Mac-Kenzie et Segr qui
lont obtenu dans le cyclotron
(1)
de Berkeley
(Californie) en bombardant le bismuth avec
des particules :
neutrons. 2
214
85
4
2
209
83
1 - Eka He Bi
Ils ont propos de lappeler astatine (At,
instable, pour rappeler son origine et sa na-
ture radioactives). Cest llment le plus no-
ble parmi les halognes; son caractre m-
tallique doit dailleurs tre trs prononc,
plus encore que celui de liode. Contraire-
ment ce dernier, il forme des sels qui ne
prcipitent pas en solution par le nitrate dar-
gent.
Cest encore par la radioactivit artificielle
que Segr et Perrier ont prpar um isotope
de llment 43. Ils lont obtenu en bombar-
dant le molybdne avec des atomes ioniss de
lhydrogne lourd (deutons), possdant une
nergie trs leve (quon leur fournit laide
dune champ lectrique intense):
(2)
43 1 42
- neutron. Mn Eka H Mo
Cet lment port le nom de technicium (Tc,
dorigine technique et non naturelle); il par-
tage les proprits chimiques du manganse
et du rhnium, entre lesquels il se trouve
dans le systme priodique.
Enfim, le dernier lment manquant, le 61,
a t galement prpar par voie artificielle,
cette fois par la fission de luranium. On sait
que ce phnomne consiste dans lexplosion
du noyau atomique de luranium sous laction
de neutrons et rupture en deux morceaux,
deux nouveaux noyax plus petits et eux m-
(1)
Le cyclotron est un appareil gigantesque, dans
lequel on produit des particules (ions dhydrogne,
rayons a etc.) dnergie trs leve et qui servent
provoquer des transformations radioactives.
(2)
Le neutron est une particule lmentaire ayant
la mme masse que lhydrogne mais qui ne peut pas
se dcomposer en un ion et un lectron.
240
mes instables, radioactifs, qui se dcomposent
leur tour avec mission de neutrons, rayons
et en donnant dautres atomes radioactifs
etc., jusqu ce quon sarrte des noyaux
stables. Au cours des travaux sur la bombe
atomique on a identifi des dizaines et (les di-
zaines disotopes radioactifs qui se forment
par suite de la fission, et plusieurs de ces iso-
topes appartiennent llment 61. Les pro-
prits chimiques de celui-ci nont pas encore
t dcrites, mais comme il se trouve au mi-
lieu du groupe des terres rares, il partage
certainement toutes leurs proprits. Les jeu-
nes chimistes, Marinsky et Glendenir propo-
sent de lappeler par promthum (Pm). Ils
entendent par cela voquer linvention du feu
par le hros mythologique Promthe et rap-
peler aux hommes que la libration de lner-
gie atomique, utilise pacifiquement, devrait
constituer une source de bien-tre de progrs
rel.
Cest ainsi qu lheure actuelle tous les
trous du systme priodique sont remplis
soit par des lments trouvs dans la nature
soit par des lments cres artificiellement.
Nous avons vu le rle considrable que la ra-
dioactivit a jou pour atteindre ce rsultat
remarquable. Mais ce nest pas tout. Da-
prs ce que nous avons dit, il ne peut y avoir
plus de 92 lments entre lhydrogne et lu-
ranium. Ceci nexclut cependant pas la
possibilit dexistence dlments plus lourds
que luranium, cest dire qui ont des num-
ros atomiques plus levs. En effet, encore
en relation avec les travaux sur la fission de
luranium, on a prpar artificiellement 4 nou-
veaux lments, transuraniens, qui se for-
ment successivement partir de luranium,
lorsque celui-ci est soumis laction de di-
vers projectiles atomiques. Ces lments qui
portent donc les numros 93 96 sappellent:
neptunium (Np), plutonium (Pu), amricium
(Am), et curium (Cm). Cest surtout le plu-
tonium qui a acquis une importance extraor-
dinaire par suite de sa proprit de subir,
comme luranium, la fission nuclaire. Il a
t utilis comme explosif dans une des
deux bombes atomiques lances en 1945 au

Vol. I, Fasc. 8 GAZETA DE F S I CA Julho, 1948
parer les 5 lments les uns des autres par
une srie doxydations et de rductions; dans
ces oprations, luranium soxyde plus facile-
ment pour passer ltat de composs de va-
lene VI, le curium se rduit plus facilement
pour prendre la valence III et les 3 autres
ont un comportement intermdiaire.
Pour terminer, signalons quon a pu dce-
ler la prsence de quantits extrmement fai-
bles de plutonium dans les minrais de lurane
(1 part sur 10
14
). La dtection dune quan-
tit aussi faible na t possible que grce
une connaissance profonde des proprits chi-
miques du nouvel lment.
M. HASSINSKY
INSTITUT DU RADIUM, PARIS
Japon. Les amricains lui ont port un intrt
particulier; um grand nombre de ses compo-
ss a t prpar et tudi. Les proprits
des 3 autres lments ont t galement exa-
mines suffisamment (mais peu divulgues)
pour pouvoir les caractriser du point de vue
chimique.
Ces proprits sont trs proches de celles
de luranium. Comme celui-ci, les 4 lments
prsentent plusieurs degrs de valence: III,
IV, V e VI, mais tandis que la valence la
plus stable de luranium est VI, la stabilit
des nouveaux lments se dplace vers les
valences infrieures, de sorte que les compo-
ss du curium sont plus stables ltat de
valence III. Cette diffrence permet de s-
9 . HIST RIA E A N TO LO G IA
F R ED ERI C O P A SC H EN
estudos sobre a radiao espectral do corpo
negro; a realizao do galvanmetro que
tem o seu nome e o estudo da espectrografia
do infra-vermelho; estudou o efeito Zeeman,
estruturas hiperfinas, spin nuclear, etc.; em
1908 descobriu uma srie de riscas do es-
pectro do hidrognio, na regio do infra-ver-
melho e estabeleceu a frmula dessa srie
que tem o seu nome; estudou a estrutura
hiperfina do hlio e em 1919 classificou as
130 sries do espectro do neon.
Perseguido pelo nazismo foi demitido em
1933 sendo-lhe permitido trabalhar numa pe-
quena dependncia em Reichsanstalt.
Se bem que condicionado pela situao em
que se encontrava, continuou os seus estudos,
fazendo vrias publicaes.
A Gazeta de Fsica s hoje, passados quse
dois anos e devido situao internacional,
teve conhecimento da morte de Frederico
Paschen e portanto s hoje presta publica-
mente a sua homenagem ao cientista e ao
homem.
M. H. S.
241
Morreu Frederico Paschen em Potsdam,
no dia 20 de Fevereiro de 1947, com 82
anos de idade, depois de ter dedicado a me-
lhor parte da sua vida investigao cien-
tfica.
Esprito muito vivo, trabalhador incansvel,
comunicando aos seus discpulos um grande
entusiasmo por todos os trabalhos a que se
dedicava, dispunha todos os dias de uma hora
para discutir com os estudantes, problemas
cientficos.
Duma grande honestidade, a sua modstia
levava-o a esquecer o grande papel que de-
sempenhava no desenvolvimento cientfico dos
que com ele trabalhavam; louvava e entu-
siasmava todos os trabalhos honestos, mas
no se cansava de destruir e criticar os resul-
tados falseados ou feitos com menos hones-
tidade.
Era Foreign Member of the Royal Society
e Honorary Fellow of the Physical Society
e foi o Presidente da Physikalisch Tech-
nische Reichsanstalt.
Entre os seus trabalhos podemos citar os

Vol. I, Fasc. 8 GAZETA DE F S I CA Julho, 1948
SO B R E O O B J E C TO , M TO D O E E STU D O D A F SI C A
Da Introduo ao livro do Prof. Harley
Howe (Cornell University), Introduction to
Physics, recentemente aparecido em segunda
edio, extramos algumas passagens que,
pela sua simplicidade e rigor, nos parece de
todo o interesse dar a conhecer.
. . . . . .
O objecto da Fsica
Todos os conhecimentos cerca da matria
viva ou no viva podem agrupar-se dentro da
biologia e da fsica.
. . . . . .
Embora a fsica, no seu sentido mais lato,
inclua todo o conhecimento da matria no
viva usual limitar os tpicos incluidos num
curso formal queles fenmenos que habitual-
mente associamos mecnica, calor, som,
luz, electricidade e magnetismo.
. . . . . .
A observao casual de muitos fenmenos
classificados como fenmenos fsicos mostra-
nos a matria em movimento sob a aco de
qualquer fora exterior.
. . . . . .
Encontram-se corpos capazes de, em certas
circunstncias, pr outros corpos em movi-
mento, isto , realizar aquilo a que o fsico
chama trabalho; diz-se que tais corpos pos-
suem energia. O fsico observa que determi-
nada variao de energia se produz prtica-
mente em todos os fenmenos estudados.
Portanto podemos dizer que o fsico estuda
a matria, o movimento, a fora e a energia.
. . . . . .
Quando alguma ocorrncia observada na
natureza analisada at ao ponto de se veri-
ficar que ela ilustra um princpio geral pr-
viamente reconhecido, ou lei, da matria,
movimento, fora ou energia, o fenmeno
considera-se ento explicado, pelo menos na
medida em que a fsica susceptvel de ex-
242
plicar. Tais explicaes constituem a parte
principal dum curso padro de introduo
fsica.
. . . . . .
Um estudante que vai para
.
uma Escola
estudar fsica ouvir dizer, provvelmente
muitas vezes, que a matria pode ser consti-
tuida por vrias substncias; que uma poro
de substncia pura pode ser dividida em mo-
lculas, todas iguais;...
. . . . . .
Mas ouvir dizer estas coisas (vagas ou
muito gerais) no satisfaz o estudante; ele
pretende, como estudante, saber por que que
as informaes que lhe do so consideradas
verdadeiras. Qualquer apreciao real das
razes deve apoiar-se sobre o conhecimento
de como os cientistas tm aplicado as leis
fundamentais da fora e movimento inter-
pretao das suas observaes da matria.
Informar a respeito destas questes deve ser
a finalidade de qualquer curso de fsica, se
esse curso pretende ser mais do que mero
repositrio de factos e se procura apresentar
uma anlise dos factos que familiarize o es-
tudante com a aplicao do mtodo cientfico
no campo dos fenmenos fsicos.
O mtodo cientfico
Compreende essencialmente 5 espcies de
actividade
1. Observao dos fenmenos.
2. Classificao das observaes.
3. Formulao de proposies gerais (cha-
madas leis) que descrevem os fenmenos
de um modo geral e de teorias que expli-
cam um grupo de fenmenos relacionados
em termos de uma lei mais simples ou mais
geral.
4. Previso do que acontecer em condi-
es supostas, baseada nas leis e nas teorias.

Vol. I, Fasc. 8 GAZETA DE F S I CA Julho, 1948
5. Experimentao, que a comprovao
da veracidade da previso e que , portanto,
a verificao da verdade da Teoria. (O con-
trole pela experincia conduz muitas vezes
a uma nova formulao da lei ou princpio).
Recomenda-se sempre ao estudante que
tenha bem presente os passos inerentes ao
seu estudo e que seja capaz, em qualquer
discusso, de saber se est observando, clas-
sificando, generalizando, prevendo, ou veri-
ficando uma teoria. No livro Como estudar,
F. Sanford diz a respeito do mtodo cient-
fico: Pode utilizar-se melhor no estudo da
fsica do que no estudo de qualquer outra
cincia. esta a razo por que a fsica se
tem, desenvolvido mais rpidamente do que
qualquer outra cincia. tambm a razo
por que o -vontade no uso do mtodo cien-
tfico se pode adquirir mais fcilmente estu-
dando fsica do que qualquer outro assunto.
. . . . . .
Auto inquirio no estudo
. . . . . .
A mera memria de factos no os torna
utilizveis nem no pensamento nem na aco.
No basta conhecer; deve-se compreender,
isto , apreciar as ligaes entre factos co-
nexos. Uma tal compreenso surge mais
fcilmente num estudante que mantm uma
actividade inquiridora em relao ao seu livro
de texto.
. . . . . .
Quem estabelea o hbito de procurar rela-
es claras entre qualquer facto novo e o seu
conhecimento prvio achar-se- livre da maior
parte do esforo de memorizao.
. . . . . .
O estudo da fsica oferece uma oportuni-
dade invulgarmente boa para a prtica de
relacionao do saber novo com a experin-
cia velha....
1 0 . Q U M I C A
A A C STI C A E A Q U M I C A
Os sons quando tm frequncias superiores
a um determinado limite deixam de ser aper-
cebidos pelo ouvido humano. Para alguns
indivduos sse limite 16000 para outros
20000 ciclos por segundo. Os sons de fre-
quncia superior chamam-se ultra-sons ou
super-sons. O ramo da Fsica que estuda
este assunto ainda a Acstica mas pode to-
mar o nome de ultra-acstica.
As frequncias ultra-sonoras, ou super-s-
nicas, podem ser produzidas mecnicamente
como os sons audveis. Por este meio, em
1876, Koenig conseguiu subir at frequncias
de 90000 ciclos por segundo.
Em 1926, Wood e Loomis trabalhavam j
no campo dos 200 a 500 quilociclos por se-
gundo; e comunicavam que esses ultra-sons
podiam produzir emulses, coagular precipi-
tados, formar nevoeiros, destruir os corpus-
culos do sangue, matar bactrias, coagular a
albumina do ovo, etc.
Reparou-se que as frequncias de vibrao
mecnica eram da ordem das frequncias elc-
tricas utilizadas em radio-comunicao e, por-
tanto, que se poderia gerar ultra-sons por via
elctrica, se se soubesse transformar impulsos
elctricos em mecnicos da mesma frequncia.
Este problema foi resolvido com a descoberta
da piezo-electricidade. Uma lmina de quar-
tzo, talhada convenientemente em relao
aos elementos cristalogrficos do cristal, goza
da propriedade (geral para os cristais hemi-
dricos) de ficar com as faces carregadas de
electricidade quando sujeita a uma traco ou
compresso e, inversamente, de sofrer uma
deformao correspondente quando se comu-
nicam s faces cargas elctricas.
Este quartzo-piezoelctrico, de maravilho-
243

Vol. I, Fasc. 8 GAZETA DE F S I CA Julho, 1948
sas aplicaes, bem conhecido do ponto de
vista prtico dos rdio-emissores amadores,
como estabilizador da frequncia de emisso,
constitue a chave de produo de ultra-sons
de muito alta frequncia.
As aplicaes dos ultra-sons so numero-
sssimas. Citarei rpidamente algumas das
mais interessantes:
Produzem emulses estveis de substncias
to imiscveis como a gua e o mercrio.
Desempoeiram os gases. Um lquido atraves-
sado por ondas super-snicas actua em relao
luz como uma rede de difraco. Modificam
a velocidade de solidificao e corresponden-
temente a estrutura metalogrfica das fundi-
es em metal; simultneamente expulsam os
vapores que estejam retidos, de modo a evi-
tar os chchos de fundio.
As aplicaes Qumica so to nume-
rosas e importantes que se pode considerar
desde j um novo ramo da Qumica-Fsica
a Sono-Qumica.
H, principalmente, dois campos de grande
intersse na Sono-Qumica:
1) A investigao de propriedades molecu-
lares dos fluidos, pela medio da velocidade
de propagao das ondas ultrasnicas de fraca
intensidade; e
2) O estudo das reaces qumicas provo-
cadas ou aceleradas por radiao ultrasnica
intensa.
Ao primeiro grupo, que no nos interessa
tanto por agora, pertencem as determinaes
de peso molecular, volume molecular, varia-
o do grau de associao molecular com a
temperatura, compressibilidade adiabtica dos
gases e razo dos calores especficos a pres-
so constante e a volume constante (para os
gases), etc.
Estas determinaes so possveis porque
as propriedades moleculares correspondentes
so funo da estrutura e esta deve afectar a
velocidade de transmisso do som, dado que
ela se faz por impulso mecnica de mol-
cula a molcula.
A velocidade de propagao do som nos
lquidos pode determinar-se com suficiente
preciso por meio de um interfermetro ultra-
244
-snico. A escolha de frequncias ultra-so-
noras determinada pelo facto de que o com-
primento de onda, para um certo lquido,
da ordem de um metro para a frequncia de
um quilociclo e desce a um milmetro quando
a frequncia sobe a um megaciclo por segundo.
No aspecto que mais nos interessa, de mo-
mento, focar o das reaces ultra-snicas
temos por exemplo a aco da radiao ultra-
-snica intensa na gua carregada de oxignio
com formao de gua oxigenada, a diminui-
o da viscosidade nas solues de gelatina,
a reduo do cloreto mercrico por oxalato,
o reagrupamento estrutural da benzazida, etc.
Uma reaco que se d com rapidez nestas
condies a da libertao do iodo duma solu-
o de iodeto de potssio contendo uma pe-
quena quantidade de tetracloreto de carbono.
Aparentemente o mecanismo da reaco con-
siste na rutura da ligao cloro-carbono e oxi-
dao subsequente do iodeto por esse cloro
livre.
Outro efeito muito interessante o da des-
polimerizao de polmeros elevados; por
exemplo, o pesa molecular do polistereno,
quando dissolvido em tolueno e submetido
aco de radiaos ultrasnicas desce de
300000 para 40000 em duas horas; d-se ini-
cialmente uma rpida degradao mas o sis-
tema evolue rpidamente para um estado de
equilbrio.
Supe-se que a fractura das ligaes qu-
micas resulte ou da frico entre as macro-
molculas do polmero e as molculas do dis-
solvente ou da frico que acompanha a
cavitao (formao e destruio de pequenas
bolhas no lquido, nas regies da onda em ra-
refaco).
A casa Eimer & Amend, E U A, fornece
um aparelho comercial que trabalha com um
comando de botes, como o de um aparelho
de telefonia, com as quatro frequncias de
400, 700, 1000 e 1500 quilociclos por segundo.
So estas as frequncias que se revelam mais
fecundas para a aplicao e pesquiza das
aces provocadas pelos ultra-sons; mas uma
pequena modificao no desenho do oscilador

Vol. I, Fasc. 8 GAZETA DE F S I CA Julho, 1948
O estudante de fsica poder ver:
Actualits scientifiques et industrielles, 522
e 523 proprits pizo-chimiques, physiques
et biophysiques des ultra-sons. Hermann;
Paris; 1937.
Supersoni cs Robert W. Wood; Brown
University Press; 1939.
Ultrasonics Ludwig Bergmann (traduo
inglesa do alemo) John Wiley & sons.
Este artigo foi respigado, em grande parte,
de Journal of Chemical Education, 25, N. 1,
1948. Laboratory, 16, N. 2.
FERNANDO NEVES DA SILVA
LICENCIADO EM CINCIAS FSICO-QUMICAS
elctrico e a troca dos quartzos pode permi-
tir o trabalho a outras frequncias. O dispo-
sitivo consta de duas partes, uma que o
gerador elctrico e outra que um vaso-labo-
ratrio contendo o quartzo num banho de
leo. Os impulsos elctricos de alta frequn-
cia gerados pelo oscilador so comunicados
por meio dum cabo ao vaso-laboratrio onde
o quartzo responde mecnicamente. A osci-
lao inicialmente amplificada de modo que
o ultra-som pode ser gerado com 500 watt.
O leitor interessado e no iniciado pode
ler com proveito o livro Les ultrasons da
coleco Que sais-je ?.
P R O B L EM A S D E EX A M E S U N I V E R SI T R I O S
F. C. L. Curso geral de Qumica e Curso de Qumica
F. Q. N. Julho, 1948.
77 Uma substncia orgnica deu anlise os
seguintes resultados: 1,000 g da substncia deram
1,169 g de CO2 e 0,478 g de OH2. A densidade do
seu vapor em relao ao ar, 3,911. 0,250 g da subs-
tncia mineralizada segundo Carius do 0,635 g de
ClAg. a) Indique a frmula molecular da substn-
cia. Escreva as frmulas de constituio dos com-
postos que podem corresponder a essa frmula global,
e d nomes aos diversos ismeros possveis. b) Inter-
prete por esquemas a hidrlise de cada um desses
ismeros. R: Frmula emprica: C3H6Cl2. Peso
molecular: M = 28,93,911=113. Nomes dos ismeros
possveis: dicloro-1-propano; dicloro-2-propano; di-
cloro-1,2-propano e dicloro-1,3-propano. Estes, por
hidrlise, do respectivamente: propanal, propanona,
propano-diol-1,2, propano-diol-1,3.
78 Qual dever ser a composio centesimal de
uma mistura de hidrolite e oxilite, para produzir,
pela aco da gua, a mistura gasosa detonante?
R: 2O2Na2 + 2OH2 4OHNa + O2; H2Ca + 2OH2
(OH)2Ca + 2H2. Mistura detonante: O + H2 OH2.
Note-se primeiramente que O2Na2=78 g de oxilite +
+ H2Ca/2=21 g de hidrolite, do oxignio e hidrognio
nas propores da mistura detonante e correspondem
aproximadamente composio centesimal. Resolvendo
o sistema x+y=100; v=x11,2/78; v=y22,421;
v=2 v, em que x e y representam os pesos de oxilite
e hidrolite, e v e v, os volumes de oxignio e hidrognio,
obtm-se a composio pedida: x=78,8 g%0 de oxilite e
y=21,2 g% de hidrolite.
79 Uma substncia aliftica A, com reaco
cida, deu anlise os seguintes resultados: 0,232 g
da substncia deram 0,528 g de CO2 e 0,216 g de
OH2. 0,335 g do seu sal de prata deixaram por cal-
cinao uni resduo de 0,162 g. Por descarboxilao
da substncia A com cal sodada, forma-se uma subs-
tncia B, voltil, cuja densidade de vapor em relao
ao ar 2,5. Substituindo um tomo de hidrognio
de B por um tomo de Cl, forma-se um nico deri-
vado clorado. Frmulas e nomes de A e B. R: Peso
molecular da substncia A, calculado a partir do seu
sal de prata: M=116. Frmula molecular: C6H12O2.
Peso molecular da substncia B: M=28,92,5=72,25.
C6H12O2 CO2 C5H12 (M1=72). A substncia B o
dimetil-propano, e a substncia A, um hexanico.
80 Uma substncia orgnica A deu anlise os
seguintes resultados: C 58,536 %; H = 7,317 %.
0,2050g da substncia deram, pelo processo de Dumas,
56 cm
3
de azoto, p. t. n. Por hidrogenao do referido
composto obtm-se uma monamina B, cujo cloropla-
tinato fornece, por incinerao, 38,800 % de resduo.
A mesma monamina, tratada por cido azotoso, d
uma substncia C. Escreva as frmulas racionais
das substncias A, B e C, e traduza por esquemas
as duas transformaes acima apontadas. R: Fr-
mula emprica da substncia A: C2H3N. Peso mole-
cular da monamina, calculado a partir do peso molecular
do cloroplatinato: 45. Frmula racional da monamina:
CH3CH2NH2. Idem para a substncia A: CH3CN;
e para a substncia C: CH3CH2OH.
Resolues de ALICE MAIA MAGALHES
245

Vol. I, Fasc. 8 GAZETA DE F S I CA Julho, 1948
I. S. T. Qumica Orgnica I Junho de 1948.
81 100 bactrias estavam presentes numa certa
cultura. 3 horas aps contaram-se 280 bactrias.
Admitindo que existe proporcionalidade entre a taxa
de crescimento e o nmero de bactrias presentes,
ache a lei de crescimento e determine ao fim de
quanto tempo a cultura produz 1.000.000 de bactrias.
R: dx/dt=Kx; x=Ce
kt
; K,C=constantes; x=nmero
de bactrias existentes no instante t. 100 = C.e
0.k
;
280=C.e
3t
, C=100, K=0,343. Lei de crescimento na-
tural: x=100e
0,343t
; aplicao numrica: x=10
6

t=30 horas.
I. S. T. Qumica Orgnica II Janeiro de 1948.
82 Numa nitrao em que se pretende trabalhar
com um D.V.S=3,5, deseja-se utilizar um cido
sulfo-ntrico com 8%. de OH2. O cido residual, aps
a recuperao dos produtos de nitrao, dever ter a
seguinte composio: SO4H2 85 %;

NO3H 2 %;
OH2 13%. Qual a composio do cido sulfo-ntrico?
R: D.V.S =
reagentes reaco
2 2
2 4
OH OH
H SO


cido sulfo-ntrico: cido residual:
SO4H
2
x SO4H2 85 z
NO3H y NO3H2 2 z
OH2 8 OH2 13 z
100 100 z
z x
z
x
y x 85 5 , 3
13 8
92


z = 0,7 x = 60 (% SO4H2) y = 32 (% NO3H)
Resolues de CHAGAS ROQUETTE
PO NTO S DE EXA M ES DO C URSO C O M PLEM ENTA R DE C INC IA S
Liceus de Lisboa Junho de 1948 1.

chamada.
33 I) A anlise qumica de 5,2 gramas de um
bicido orgnico saturado deu os seguintes valores:
carbono, 1,8 gramas; oxignio, 3,2 gramas; hidrog-
nio 0,2 gramas. Verifica-se tambm que 2,6 gramas
dessa substncia, depois de dissolvida em gua, foram
neutralizados por 25 cm
3
dum soluto binormal duma
base. Responda s seguintes perguntas: 1.) quantos
equivalentes-grama do cido entraram na referida neu-
tralizao? 2.) qual a frmula molecular do cido?
3.) qual a sua frmula racional? R: 1.) Tantos quan-
tos os equivalentes-grama da base. 1000 cm
3
:2=25:n
donde n = 0,050 equivalentes-grama; 2.) Clculo do
equivalente-grama do cido; 0,050:2,6 g=1:e, donde
e=52 g; peso molecular: 104. Frmula emprica do
cido: 1,8 C:12+3,2O:16+0,2H: que d C3O4H4 que
tambm a frmula molecular. 3.) COOH.CH2.COOH.
34 II) Tema de desenvolvimento: Hidrlise dos
Sais. Desenvolva o assunto de acordo com as seguintes
alneas: a) que entende por hidrlise; b) por que mo-
tivo que certos sais, em soluo aquosa, manifestam
propriedades alcalinas. Interpretao deste fenmeno
por intermdio da hidrlise escolhendo um desses sais
para servir de exemplo.
Liceus de Lisboa Junho de 1948 2. chamada.
35 I) Juntam-se 200 cm
3
dum soluto dum cido
com meio litro dum soluto alcalino. Os factores de
246
normalidade destes solutos so, respectivamente, 1,42
para o cido e 0,8 para o alcalino. Responda s se-
guintes perguntas: 1.) O soluto resultante da mistura
dos dois solutos anteriores, manifesta propriedades
cidas ou alcalinas? D a resposta apresentando n-
meros que a justifiquem completamente. 2.) Como
procederia se quisesse acabar de neutralizar o soluto
resultante a que se refere a pergunta anterior?
3.) Supondo que o cido de que se trata era o cido
aztico, exprima em percentagem a concentrao da-
quele soluto cujo factor de normalidade se disse ser
1,42. (Pesos atmicos: H=1; O=16; N=14). R: 1)
O soluto cido contm 0,2001,42=0,284 equivalentes-
-grama e o soluto alcalino contm 0,5000,8 = 0,4.
O soluto resultante alcalino porque 0,4>0,284. 2)
Juntaria 0,4 0,284 = 0,116 equivalentes-grama dum
cido. 3) Como NO3H=63, o soluto conteria num litro:
1,4263=89,46 g/l que equivale a 8,946 %.
36 II) Tema de desenvolvimento: Metameria.
Desenvolva o assunto de acordo com as seguintes
alneas:
a) em que consiste a metameria;
b) como se explica esse fenmeno;
c) como que as frmulas dos compostos podem
distinguir os casos de metameria;
d) exemplos de compostos que sejam metmeros
e indicao das respectivas frmulas que os distin-
guem entre si.
Resolues de RMULO DE CARVALHO

Vol. I, Fasc. 8 GAZETA DE F S I CA Julho, 1948
1 1 . A FSIC A N A S SUA S A PLIC A ES
L A C O U STI Q U E D ES SA LLES
a) Generalits
Jusquil y a une trentaine dannes, on nat-
tachait gure dimportance la question qui
fait lobjet de cet expos. Sans doute, la-
coustique thorique pure suscitait lintrt d-
minents physiciens; mais, lacoustique techni-
que et ici nous pensons surtout celle des
constructions tait trs nglige.
Les premires recherches qui dbordrent
les cadres troits de lacoustique gomtrique,
remontent 1853. Elles eurent comme auteur
un mdicin amricain, J. B. Upham. Cellui-ci
plaide en faveur dune investigation scienti-
fique dans ce domaine et il formule le re-
gret que les architectes et les chercheurs ne
sintressent pas assez une question aussi
importante.
A la suit, il y eut les tentatives hsitantes de
quelques exprimentateurs, puis, les publica-
tions plus importantes de Henry, Smith,
Tyndal, Rayleich et autres dont les travaux
ont fray la voie louvrage rsolument
novateur de W. C. Sabine.
Ce dernier, professeur lUniversit Har-
vard, commena, ds 1895, ltude scientifi-
que et systmatiquement pousuivie, de lacous-
tique des salles, au point de vue, surtout, de
la rverbration. Les dcouvertes furent ti-
midement appliques par quelques architectes,
avec un succs persistant. Pendant longtemps,
louvrage de W. C. Sabine resta la seule
base vraiment solide pour lapplication imm-
diate de lacoustique la construction et au
revtement des auditoires. Mais, il arrivait en-
core trop souvent quon ngliget les donnes
que lexprimentation lui avait fait dcouvrir.
Pourtant, les travaux du prcurseur Sa-
bine ont port des fruits. Aprs la premire
guerre mondiale, lacoustique des construc-
tions est devenue une science importante, une
science promise des dveloppements de plus
en plus considrables, vu le nombre, des pro-
blmes thoriques et pratiques que posent les
exigences toujours plus imprieuses de la vie
moderne.
Lobjet de cet article est de montrer quil
est, dsormais, possible un architecte de
dterminer davance les qualits acoustiques
dun auditoire ou dun btiment, et, ds lors,
de calculer quels matriaux doivent tre uti-
liss dans des circonstances bien dfinies.
Dans cette estimation, il faut dabord se
prmunir contre les bruits parasites qui se
produisent aussi bien lintrieur qu lext-
rieur dune salle daudition. Il faut aussi as-
surer une audition nette et impeccable des
sons, paroles ou musique.
Les facteurs qui, cet gard, sont appels
jouer un rle dans une salle dtermine,
sont, donc, le profil et le volume du local,
sa capacit dabsorption et la valeur isolante
de ses parois.
Considrons, succinctement, ces facteurs et
voyons comment ils influent sur les qualits
dune salle.
Profil de la salle.
Dterminer quel est, pour une salle, le pro-
fil le plus avantageux cest un problme
dacoustique gomtrique. La question est de
moindre importance quand il sagit simplement
damortir les sons afin de crer une atmos-
phre de calme et, ainsi, des conditions de
travail favorables, dans un bureau ou un ate-
lier par exemple, Mais, le problme peut rev-
tir une importance extrmement grande quand
il se pose pour une salle de concerts ou de
confrence. L, en effet, on exige que cha-
que phrase musicale soit perue dans toute
la riche varit de ses nuances. Il est indis-
pensable, dautre part, que tous les auditeurs
puissent, sans effort, entendre les paroles des
orateurs.
Il va de soi, que, dans les salles de confren-
ces ou de concerts, la puissance du son doit
tre galement rpartie dans lespace entier du
local et quil faut viter toute interfrence.
247

Vol. I, Fasc. 8 GAZETA DE F S I CA Julho, 1948
Pour que le son direct et le son rflchi
se renforcent mutuellement, il faut que la
diffrence des distances parcourues par les
deux ondes nexcde pas 13 m. Si cette dis-
tance dpasse 20 m, des interfrences gnan-
tes se produisent et mme des chos.
LAlbert-Hall de Londres, par exemple,
est dot dun profil si mal encontreux, cause,
sourtout, de sa haute vote circulaire, quil
est impossible dy goter nimporte quelle
musique. Par contre, lexcellente acoustique
de la salle Pleyel de Paris, est considrable-
ment renforce par la lgre inclinaison des
parois latrales de ldifice et la forme para-
bolique de son plafond. Alors quantrieure-
ment on stait souci seulement du profil du
btiment, on tint compte aussi, aprs lincen-
die de 1928, de la capacit dabsorption de
la salle, si bien quaujourdhui cette salle de
concerts rpond vraiment , toutes les exi-
geances de lacoustique.
Au sujet de la question envisage ici, on
consultera avec fruit louvrage de Bagenal
et Wood (1).
Capacit dabsorption de la salle.
Dans lacoustique des constructions, on ap-
pele Coeficient dabsorption dun panneau
la fraction de lnergie sonore incidente qui
nest pas rflchie par lui. Le capacit totale
dabsorption dun mur ou dun objet est, gn-
ralement, exprime en units de fentre ou-
verte. On admet, en effet, quune fentre
ouverte laisse passer, donc absorbe, la tota-
lit du son.
Ainsi, par exemple, un mur de 20 m
2
cou-
vert dun revtement absorbant dont le coeffi-
cient dabsorption est de 0,40, produira une
absorption totale de 8 m
2

(1)
.
Le pouvoir absorbant dun matriau dpend
de beaucoup de facteurs. Ceux-ci influent
non seulement sur la capacit dabsorption
frquence dtermine, mais aussi sur le cours
de labsorption en fonction de la frquence.
(1)
Lorsque la surface est exprime en pieds carrs,
on donne le nom de sabine une unit de fentre
ouverte.
248
Il est, donc, ncessaire de connatre la rela-
tion qui existe entre labsorption et la fr-
quence, pour savoir si tel ou tel matriau
entre en ligne de compte pour la solution
pratique de lun ou lautre problme. Aussi,
le coefficient dabsorption dun matriau est-
-il souvent dtermin aux frquences 128,
256, 512, 1024, 2048 et 4096 Herz.
A 123 Hz, par exemple, le coefficient dune
bonne matire absorbante ne dpassera, g-
nralement, pas 0,30. Pour des frquences
plus leves, il est rare que la valeur 0,90
soit dpasse. Les matriaux de construction
ordinaires, tels le bton, la brique, le bois, le
verre, etc., ont un coefficient infrieur
0,10. Etoffes et tapis peuvent absorber de
5 60% daprs la frquence et daprs la
nature du tissu. Une personne adulte possde,
en moyenne, une valeur dabsorption de
0,40 m
2
.
La capacit dabsorption dune salle influera
sur son temps de rverbration et sur linten-
sit du son. La valeur acoustique dun local
est, donc; dtermine conjointement, par ces
deux facteurs.
Le niveau sonore est exprim en dcibel,
de la manire suivante:
. log 10 log 10
0
10
0
10


I
I
s
Dans cette formule, I et reprsentent res-
pectivement lintensit et la densit dnergie
perue. I
0
et
0
, les mmes grandeurs la
limite daudibilit dune oreille normalle. (Pour
1000 Hz,
0
=2,610
20
Joules/cm
3
et I
0
=910
6

watt/cm
2
).
A titre dillustration, voici les moyennes
des niveaux de certains sons (2), (3):
Seuil de sensation: 0 db
Jardin tranquile: 20 db
Rue calme: 30-50 db
Conversation paisible: 40 db
Bruits moyens dune habitation: 50 db
Rue commerante trs frquente: 60 db
Bureau bruyant: 70 db
Rue de trs grande circulation; 75 db
Limite suprieure de loue
(sensation douloureuse): 130 db

Vol. I, Fasc. 8 GAZETA DE F S I CA Julho, 1948
Lintensit du son diffus dans une chambre
sera, ltat stationnaire, proportionnelle
la puissance de la source sonore et invers-
ment proportionnelle labsorption totale du
local. Si lon peut, par exemple, rendre 20
fois plus grande la capacit dabsorption dun
bureau, grce au placement dun matriau
absorbant, lintensit du bruit y diminuera de
13 db; car,
. 13 20 log 10 log 10 log 10
1
2
2
1

A
A
I
I

Rverbration. Lorsque, dans un local, une
source sonore cesse brusquement dmettre de
lnergie, les ondes peuvent continuer quelque
peu tre audibles. En effet, leur intensit
ne diminue que progressivement par absor-
ption chaque rpercussion sur les murs et
les objets environnants. Ce phnomne est
appel rverbration.
Le temps de rverbration est, par dfini-
tion, lintervalle de temps durant lequel lin-
tensit dun son non entretenu retombe jusqu
1 millionime de sa valeur durant lequel, donc,
son niveau diminue de 60 db.
Le temps de rverbration dun local d-
pendra de sa capacit totale dabsorption et
de ses dimensions. On trouve (4):

. sec./m. 071 , 0
1 log
10


K
S

V
k R


Dans cette formule R est le temps de r-
verbration, V le volume du local, S la sur-
face des parois absorbantes, , le coefficient
moyen dabsorption, k, une constante dpen-
dante de la forme du local. Dans presque
tous les problmes pratiques, k, peut tre
considr comme gal 0,071 sec./mtres
(1)
.
La rverbration dtermine, dans une trs
large mesure, la valeur acoustique dune salle.
Le temps optimum de rverbration dpendra
de lusage auquel on destine le local.
Isolation phonique
Le passage du son travers les murs
dune salle sopre principalement par les

(1)
Si est petit, la formule ci-dessus peut ais
ment se ramener celle de Sabine:
A
V
R dans
fissures et les ouvertures, par les vibrations
propres des murs et des conduites et, en une
certaine mesure, par conduction. Il importe
de noter ici que fissures et ouvertures laissent
passer une nergie sonore beaucoup plus con-
sidrable quon ne sy attendrait. Si bien que
cest souvent une aberration de raliser liso-
lation phonique lorsque le bruit peut, tout
de mme, pntrer facilement par les fentes
des parois.
Au choc dune onde sonore qui la frappe,
une paroi se met vibrer. Elle peut, alors,
faire elle-mme office dmetteur sonore et
ds lors, produire dans le local, un bruit pa-
rasite. Contre de pareilles transmissions de
son, il y a un premier procd: donner aux
murs une paisseur suffisante. En effet, la
capacit disolement exprime en db, est fonc-
tion linaire du logarithme du poids par unit
de surface du mur.
Lisolement peut tre ralis aussi par lab-
sorption du bruit dans les pores de matires
poreuses. Inutile de dire quici la capacit
disolement est directement proportionnelle
lpaisseur db la couche absorbante.
Dans les cas o il faut atteindre un trs
haut degr disolement le mieux est de com-
biner la perte par absoption dans des mat-
riaux poreux avec la perte par inertie dans
des murs en matriaux solides. On peut pro-
voquer une perte dnergie de 60 db en utili-
sant des panneaux relativement minces dont
les intervalles sont remplis de matires absor-
bantes.
Le coefficient de transmission dune paroi
est la fraction dnergie sonore incidente qui
traverse cette paroi. La transmission totale
peut, elle aussi, sexprimer en units de fentre
ouverte (units de surface), de nouveau, dans
lhypothse quune fentre ouverte laisse pas-
ser tous les sons incidents. Naturellement, la
formule de la transmission totale est T = t
n
s
n
.
Dans cette formule, Sn est la surface dont
tn est le coefficient de transmission. Il est
ais de prouver que
I
1
/I
2
=A/T
Ici, I
1
est lintensit du bruit lextrieur
du local, I
2
lintensit du son parvenu dans
249
161 , 0

laquelle A = S, est labsorption totale des parois.

Vol. I, Fasc. 8 GAZETA DE F S I CA Julho, 1948
la salle, A labsorption totale de la chambre
et T la transmission totale. La perte dner-
gie, formule en dcibel, est, ds lors:
T
A
10
log 10 d
Dans le tableau ci-dessus, on trouvera la
limite suprieure du niveau sonore qui peut
encore subsister dans des locaux dtermins
sans effets gnants.
Il y a lieu de tendre ce que lisolement
et labsorption des salles soient adapts ces
chiffres.
Studio pour prise de films ou de
disques de gramophone: 6- 8 db
Studio de radio: 8-10 db
Hpital: 8-12 db
Salle de musique: 10-15 db
Appartement: 10-20 db
Theatre, auditoire, salle de lecture: 12-24 db
Cinma: 15-25 db
Bureau particulier: 20-30 db
Btiment public: 25-40 db
b) Acoustique de diverses salles
1. Auditoires.
La valeur acoustique dune auditoire est
dtermine, avant tout, par le degr de net-
tet, ds lors, de naturel, avec lequel sont
entendues les paroles de lorateur.
La nettet de laudition, lintelligibilit se
mesure au cours dexpriences darticulation.
Des auditeurs prennent place diffrents en-
droits de la salle; ils crivent ce quils enten-
dent lorsquun orateur prononce distinctement
des syllabes et des mots sans signification.
Le pourcentage moyen des mots qui son
exactement perus est le facteur darticula-
tion du local.
Lexprience dmontre quun facteur de
85% correspond une intelligibilit excellente.
Si le facteur nest que de 65 %, laudition est
fatiguante. En dessous de 65 %, la nettet
de laudition est insuffisante. En pratique, on
admet que, pour quun auditoire soit bon, le
facteur darticulation ne peut pas tre infrieur
75 %.
250
Ce facteur dpend de la forme de la salle,
de lintensit des bruits parasites, de linten-
sit de llocution des orateurs et du temps
de rverbration. Il rsulte dobservations
faites que dans les circonstances les plus fa-
vorables, on obtient un facteur darticulation
qui nexcde jamais 0,95 (5).
Influence da profil de la salle.
Ce qui a t dit plus haut propros de la
forme dune salle reste naturellement valable
pour les auditoires, comme dailleurs pour tous
les difices o le public doit couter discours
ou musique. En gnral, on obtient une audi-
tion plus claire dans un local large et peu
profond que dans une salle troite et profonde.
Dans de trs grands auditoires, le facteur
darticulation sera toujours de 10 % inf-
rieur celui quon calcule en omettant de te-
nir compte de la forme du local. Dans de
petits auditoires, le profil peut tre choisi de
faon ne diminuer en rien le facteur darti-
culation.
Influence du bruit.
Il va de soi quun bruit parasite gne la
nettet de laudition. Dans lhypothse o
lorateur peroit aussi le bruit gnant, le fac-
teur darticulation diminuera en moyenne, sui-
vant la relation approximative:
k = (0,25) nk, n<1
Dans cette formule, n indique le rapport
du niveau du bruit (en db) celui des paroles
perues (i).
Influence du niveau sonore et de la
rverbration
Dans les conditions idales de profil, de
bruit et de rverbration, laudition la plus
claire sobtient pour un niveau du son de 70 db
(5). A vrai dire, la moyenne des orateurs
natteignent pas cette valeur, sauf avec le se-
cours damplificateurs.
Suivant les dimensions et labsorption de
lauditoire, la puissance sonore dloculation

(1)
Daprs les donnes de Knudsen (5).

Vol. I, Fasc. 8 GAZETA DE F S I CA Julho, 1948
pour la frquence 512 Hz et sans aplication,
en fonction du volume dun auditoire parfai-
tement calme et de profil parfait. Les traits
en pointill indiquent le facteur darticulation
pour une rverbration et un volume dter-
min
(1)
.
Ainsi, nous en dduisons que pour un au-
ditoire de 1250 m
3
, par exemple, laudition
la plus claire est obtenue pour un temps de
rverbration de 0,8 sec. Le facteur darti-
culation est alors, dans des circonstances
idales, 0,88. Pour une rverbration de
1,2 sec, le facteur tomberait 0,87.
Si lon admet quun facteur infrieur 0,75
ne donne plus satisfaction, la figure ci-dessus
montre que lintelligibilit sera insuffisante
dans des auditoires plus grands que 40.000 m
3
,
du moins, lorsque le son ny est pas amplifi.
Nous avons dj dit que les, rsultats com-
ments ci-dessuse concernent une frquence
de 512 Hz. Mais, il est normal quon dsire
conserver les caractristiques de llocution
naturelle dans laquelle les frquences inf-
rieures et suprieures sont moins fortement
marques. Dans ce cas, il faut faire en sorte
que toutes les frquences atteignent simulta-
nment la limite daudibilit. Les frquences
extrmes devront, donc, tre amorties plus
lentement et elles devront avoir un temps de
rverbration plus long.
Mais il y a lieu de tenir compte non seule-
ment du spectre de la parole, mais aussi de
la sensibilit de loreille humaine et de la
capacit dabsorption de lair qui joue un rle
dans le cas des hautes frquences. Ds lors,
on arrive la conclusion que le meilleur ren-
dement sera obtenu lorsque labsorption totale
de lauditoire, exprime en % de labsorption
idale 512 Hz, atteint les proportions
suivantes (2)
128 Hz: 58 %
512 Hz: 100 %
2048 Hz: 95 %

(1)
Dduite des donnes de Knudsen (5) (7).
(2)
Ces chiffres ont t dduits des donnes de
Mac Nair (6) et Knudsen (7).
251
pour un confrencier slvera une moyenne,
de 45 55 db.
Il est vident que si le temps de rverb-
ration est long, il influence dfavorablement
la clart de laudition. En effet, les sons non
encore teints troubleront ceux que produisent
les mots suivants. Envisage de la sorte, laudi-
tion la plus claire serait obtenue avec un temps
de rverbration le plus court possible. Mais
une rverbration courte exige une grande
absorption; et, celle-ci, son tour, entrane
une baisse de la lintensit du son; do,
abaissement du facteur darticulation. Il faut
donc sefforcer dobtenir un temps de rver-
bration le meilleur possible, ou, ce qui re-
vient au mme, une absorption admissible.
Il est certain quun orateur est port adap-
ter sa voix la sonorit obtenue. Dans un
grand auditoire, il parle dune vois plus forte
que dans un petit local. Lorsque, donc, on
tient compte dune part des donnes statisti-
Fig. 1
ques rassembles ce sujet, et dautre part,
de linfluence de la sonorit et de la rverb-
ration sur le facteur darticulation qui peut
aussi tre dtermin quantitativement, on est
mme de calculer la valeur optima de r-
verbration pour un auditoire de volume donn.
Dans la figure 1, le trait plein reprsente
la valeur optima du temps de rverbration,

Vol. I, Fasc. 8 GAZETA DE F S I CA Julho, 1948
2. Salle de concerts.
Il va de soi que le profil des salles de con-
certs doit tre tel quil assure une distribution
homogne de lnergie sonore. Ici aussi, il
y aura lieu de combattre autant que possible
les bruits parasites.
Nous omettons, ici la question de la meil-
leure disposition de lorchestre et des solistes.
Il y a l, aussi, dailleurs, un problme
dacoustique gomtrique
Pour dterminer le meilleur temps de rver-
bration, il faut sen remettre absolument au
jugement des musiciens. En effet: la valeur
acoustique dune salle de concert ne peut pas
tre exprime par lun ou lautre facteur nu-
mrique, comme cest, tant soit peu, le cas
pour les salles de confrences.
Une salle de concert peut trs bien tre
parfaitement approprie telle ou telle audit-
ion musicale; mais, se rvler moins adapte
lexcution impeccable dun autre concert.
Bach, par exemple, a compos sa musique en
vue de lglise S. Thomas Leipzig o le
temps de rverbration est relativement court.
Ds lors, les oeuvres de ce clbre composi-
teur nacquerront leur plein relief que dans
une salle dote des mmes caractristiques
acoustiques. Sans doute, chaque morceau peut
tre lobjet dune adaptation acoustique; et,
la chose est, dailleurs, effectivement ralise
dans un studio. Mais, pour une salle de con-
cert ordinaire, on se contentera dun temps
de rverbration moyen tabli suivant lavis
dexperts musiciens dment qualifis. Sans
doute les expriences en cours dans ce
domaine doivent encore tre poursuivies long-
temps, mais, les rsultats obtenus jusquici
donnent dj satisfaction.
En se basant sur des principes thoriques
ainsi que sur les rsultats de mesure effec-
tues dans diverses salles dEurope et des
Etats-Unis, Knudsen (8) a calcul le meilleur
temps de rverbration pour des salles de
musique. La figure 2 reproduit ce temps de
rverbration fonction de la capacit de la
salle pour une frquence de 512 Hz.
Pour les salles de concert, il est, certaine-
252
ment trs important que la musique soit perue
dans toute la richesse du spectre sonore avec
laquelle elle est mise. Si lon tient compte
du spectre moyen, labsorption totale de la
salle doit rester la mme entre 512 et 4096 Hz,
tandis quelle doit tre environ 2,5 moindre
par 128 Hz.
3. Studios
Dans les studios, le son est peru par le
microphone; si bien que lon obtient le mme
effet que lorsquon coute dune seule oreille.
De ce fait, il est impossible de concentrer la
perception sur un son venant dune direction
dtermine. Il en rsulte une rverbration
apparemment plus grande et une audition ren-
force de tous les sons, quelle que soit leur
origine. Cest pourquoi, dans un bon studio,
les bruits parasites ne peuvent pas dpasser
10 db.; tandis que le temps de rverbration
ne peut tre que les 2/3 du temps idal des
salles de musique et de confrences ordinaires.
Fig. 2
Dans les studios modernes, on recourt
des panneaux ou des draperies et, ainsi, on
peut faire varier le temps de rverbration
dans un grand intervalle (0,5 4 sec.).
4. Salles de cinma
Dans les salles de cinma, le niveau sonore
peut toujours tre port la valeur idale de
70 db. En vue damener une bonne articula-
tion, il faudra maintenir la rverbration
aussi courte que possible. Ceci est dautant

Vol. I, Fasc. 8 GAZETA DE F S I CA Julho, 1948
du rendement fourni par le personnel qui y
travaille. Ladaptation acoustique des bu-
reaux, des centrales tlphoniques et autres
locaux similaires entrane, donc, comme con-
squence, un accroissement notable de leffi-
cience des employs. Nous renvoyons,
ce sujet, aux publications, notamment, de
Laird (9), et de Wynne (10).
Pour illustrer notre expos, voici encore
Un exemple chiffr
I1 sagit dune petite salle, dune capacit de
1500 m
3
. Ses parois ont un dveloppement
de 900 m
2
. Une fraction de cette surface,
soit 80 m
2
est garnie de tapis; une autre por-
tion, soit 100 m
2
, est occupe par 300 siges
capitonns. En outre, une partie (600 m
2
)
consiste en matriau durs: bois, verre, etc
Les 120 m
2
restants sont susceptibles dtre
revtus de matires absorbantes. Il est pos-
sible de calculer quel doit tre le coefficient
dabsorption de ces matires, si lon affecte
cette salle des confrences ou des excu-
tions musicales.
Daprs la figure 1, nous constatons que le
temps idal de rverbration pour la parole
sera de 0,8 sec. dans un local de 1500 m
3
.
Comme la petite salle, dont il est question
doit tre approprie galement des sances
de musique, nous supposerons un temps de
rverbration de 1 sec. (v. fig. 2), dans
lhypothse o la salle, remplie au 2/3 de
sa capacit, est occupe par quelque 200
personnes.
La formule de rverbration


1 log
071 , 0
10
S
V
R
dans laquelle R = 1 sec., V = 1500 m
3
, et
S = 900 m
2
, nous donne =0,24 et A=S=
=216 m
2
.
Labsorption totale 512 Hz doit donc
slever 216 m
2
. Si lon dsire assurer
une audition naturelle et non dforme de la
parole et de la musique, il faut que labsorp-
tion ne dpasse pas 110 m
2
128 Hz (58 %.
pour la parole; 40% pour la musique) et ,
253
plus imprieux que la prise de sons, dans
les studios, na pas t sans provoquer une
certaine rverbration. Il importe, cependant,
de se garder de toute exagration dans ce
sens, afin de ne pas diminuer la richesse so-
nore de la musique.
Le temps idal de rverbration est repr-
sent en fonction de la capacit de la salle,
pour la frquence 512 Hz. (figure 2). Pour
viter la dformation du spectre sonore,
labsoption totale de la salle devra, pour
128 Hz, atteindre environ la moiti du pou-
voir absorbant 512 et 2048 Hz.
5. Bureaux
Il ny a pas grand intrt dterminer le
temps de rverbration idal pour des locaux
usage de bureaux. Ce qui importe surtout
ici, cest damortir le plus possible les bruits.
Ceux-ci sont produits dans la salle elle-mme
(cliquetis des machines crire, sonneries du
tlphone, etc....) ou bien, ils proviennent
de lextrieur et sont perus dans la salle par
suite du manque disolement.
Comme, dans un local clos lintensit so-
nore est, de faon approximative, invers-
ment proportionnelle labsorption totale, on
visera, quand il sagit dun bureau bruyant,
porter au maximum la capacit dabsorption.
Voici les directions qui ont, gnralement,
donn satisfaction. Si le plafond est une
hauteur de 3 m. 4,50 m. on le revtira
dun matriau absorbant coefficient moyen
(128-512-2048 Hz) de 0,30 0,50. Si la vote
se trouve une hauteur allant de 4,5 9 m.,
ce coefficient ira de 0,50 0,80. Pour des
bureaux plus hauts encore ce nest pas seule-
ment le plafond, mais aussi le parois quon
revtira de matriaux absorbants. La capa-
cit dabsorption de ces derniers restera, au-
tant que possible, la mme partir de 512 Hz
jusquaux frquences suprieures. A 128 Hz,
elle tombera 1/3 ou 1/2 (daprs Knudsen (5)).
Il y a lieu daccorder une attention parti-
culire au fait que les bruits produits aux
environs dun bureau exercent une influence
trs dfavorable sur la qualit et la quantit

Vol. I, Fasc. 8 GAZETA DE F S I CA Julho, 1948
2048 Hz, 210 m
2
(respectivement 95% et
100%).
Le calcul se fait alors suivant le schma
suivant:
20 db., le facteur darticulation diminuera
dans la mesure suivante: k=0,250,42
0,87=0,09. Le restant, soit 0,78, est en-
core suffisant.

V = 1500 m
3
S = 900 m
2

128 Hz 512 Hz 2048 Hz
Temps de rverbration . . . . . . . . . . . . . . . 1 sec
Coefficient moyen . . . . . . . . . . . . . . . . . . . 0,24
Absorption exige: A = S . . . . . . . . . . . . . . 110 m
2
216 m
2
210 m
2

Absorption Coeff. Totale Coeff. Totale Coeff. Totale
Tapis: 80 m
2
. . . . . . . . . . . . . . . . . . . . . . . 0,25 20 m
2
0,45 36 m
2
0,45 36 m
2

Matriaux durs: 600 m
2
. . . . . . . . . . . . . . . 0,04 24 m
2
0,04 24 m
2
0,04 24 m
2

Personnes: 200 . . . . . . . . . . . . . . . . . . . . . 018 m
2
36 m
2
0,40 m
2
80 m
2
0,47 m
2
94 m
2

(p. p. = par personne) . . . . . . . . . . . . . . . p. p. p. p. p. p.
Siges capitonns: 100 . . . . . . . . . . . . . . . . 014 m
2
14 m
2
0,22 m
2
22 m
2
0,22 m
2
22 m
2

(p. s. = par sige) . . . . . . . . . . . . . . . . . . p. s. p. s. p. s.
Absorption totale dj ralise . . . . . . . . . . . 94 m
2
162 m
2
176 m
2

Reste absorber . . . . . . . . . . . . . . . . . . . . 16 m
2
54 m
2
34 m
2

Coefficient correspondant par 120 m
2
. . . . . . 0,13 0,45 0,28

Si la surface de 120 m
2
,

reste libre, est
compltement garnie de matriau absorbant,
celui-ci devra, donc, avoir le coefficient sui-
vant: 128 Hz:0,13 512 Hz:0,45;
2048 Hz:0,28.
Mais il est possible que, par manque de
matriau, ou par souci dobtenir une rparti-
tion homogne du son, sans cho ni interf-
rence, il soit ncessaire ou avantageux de ne
revtir quune partie de la surface reste
libre. Supposons, par exemple, quon recou-
vre seulement 70 m
2
,

de telle sorte que la
surface totale en matriau dur soit rame-
ne 650 m
2
.

Dans ce cas, le coefficient
dabsorption devra avoir, pour les trois fr-
quences, les valeurs respectives de 0,20, 0,74,
et 0,46.
Si la salle est parfaitement tranquille et
si la rpartition du son est homogne, le fac-
teur darticulation, pour R=1 sec. sera den-
viron 0,87 (voir figure 1).
Si les paroles prononces sont perues avec
une puissance de 48 db., par exemple (qui
sera atteinte par un orateur moyen dans pa-
reille salle) et si les bruits parasites atteignent
254
REFERENCES
(1) BAGENAL and WOOD Planning for
good acoustics; Methuen, 1931.
(2) R. H. GALT Journ. Acous. Soc. Am.;
2, 30, 1930.
(3) R. S. TUCKER Journ. Acous. Soc.
Am.; 2, 59, 1930.
(4) C. F. EYRING Journ. Acous. Soc.
Am.; 1, 217, 1930.
(5) V. O. KNUDSEN Architectural Acous-
tics; New-York, London, 1932.
(6) W. A. MAC NAIR Journ. Acous. Soc.
Am.; 1, 242, 1930.
(7) V. O. KNUDSEN Journ. Acous. Soc.
Am.; 1, 58, 1929.
(8) V. O. KNUDSEN Journ. Acous. Soc.
Am.; 2, 434, 193t.
(9) D. A. LAIRD Journ. of Industr. Hy-
giene; 9, 10, 1927.
(10) S. W. WYNNE Journ. Acous. Soc.
Am.; 2, 12, 1930.
P. MARIENS
CENTRE D

ETUDE SCIENTIFIQUE
ET TECHNIQUE DU FROD, LOUVAIN

Vol. I, Fasc. 8 GAZETA DE F S I CA Julho, 1948
12. INFO RM A ES V A RIA S
en micelas de gran tamao dispersas en un lquido
istropo, por exemplo agua, y hgase fluir por un
tubo horizontal de unos 2 mm de dimetro, para lo
cual se ejerce presin por un, extremo mediante una
pera de goma. Al cesar la presin, debiera continuar
el movimiento a causa de la enercia, y cesar rpida-
mente por efecto de la viscosidad. Lejos de ser as,
se observa un retroceso brusco que puede llegar a ser
de algunos centmetros. Tan raro fenmeno ha encon-
trado una aplicacin no menos curiosa. Utilizando la
secrecin cervical de las vacas se puede averiguar
cundo se hallan en condiciones propicias para ser
fecundadas. En la secrecin cervical de la mujer, el
retroceso, que es de 1 a 9 milmetros en condiciones,
normales, pasa a ser de 25 a 30 mm en el momento
de la ovulacin.
Fruto de las actividades del Club Britnico de
Reologa es el libro en cuestin. Basta transcribir su
ndice para que se vea la importancia de la nueva
ciencia y el amplio campo de sus aplicaciones.
I. Reologa de los metales, polmeros y lquidos.
II. Relaciones entre los ensayos de comprsion y
cizalladura.
III. De cmo cambian com el tiempo las tensiones
y las deformaciones.
IV. Nomenclatura y sbolos.
V. Aplicacin de la reologa a la medicina.
VI. La reologa en las Bellas Artes.
Merece citarse tambin un apndice destinado al
anlisis exponencial de curvas experimentales, que ser
de utilidad siempre que se trate de desarrollar una
funcin del tiempo en suma de exponenciales. J. P.
High Resolution Spectroscopy
por S. Tolansky
Methuen & Co, Ltd, London, 36 Essex Street, Strand
W. C. 2.
Neste livro props-se o autor fazer numa descrio
detalhada da tcnica experimental da espectroscopia
moderna de elevada resoluo, conseguindo completa-
mente realizar o seu objectivo.
O livro consta de 15 captulos. No primeiro captulo
faz-se um estudo da largura das riscas espectrais,
assunto de fundamental importncia para quem se
dedique a estudos espectrogrficos, quer no domnio
dos Raios X e gama, quer no domnio do visvel. Em
seguida o autor fez um estudo das origens luminosas
empregadas, seguindo-se uma descrio exaustiva dos
diferentes interfermetros, assunto a que dedicada
pelo autor, particular ateno. Finalmente; aps al-
gumas noes fotogrficas, termina por um captulo
255
CRITICA DE LIVROS
Essais in Rheology.
Publicado por el Club de Relogos britnicos. Lon-
dres, Sir Isaac Pitman and Son Ltd.
Es muy probable que la mayor parte de los lecto-
res pasen por alto el ttulo de este libro creyendo que
no les atae. I si alguien se detiene en l, ser porque
sienta curiosidad ante lo extrao del vocablo elegido
para denominarlo, curiosidad que se avivar al ver que
el subttulo reza as: A contribution to the litera-
ture of a new science.
Y, en efecto, se trata, por decirlo as, de la partida
de nacimiento de una nueva ciencia, cosa singular
puesto que haba motibos para pensar qne todos los
conocimientos humanos estaban ya perfectamente cla-
sificados y que cualquier nueva adquisicin habra
de encajar en alguna de las disciplinas que ya se cul-
tivan en los centros cientficos.
Surgi la novedad cmo consecuencia del desarrollo
de la nueva industria de materias plsticas, pero en
los pocos aos qu lleva de vida su campo de accin
se ha extendido a los campos ms diversas de la tec-
nica, desde los lubricantes hasta la ciruga. Adems,
si bien naci por razones tecnicas, todo induce a creer
que ha de tener importancia terica, pues plantea
problemas que obligan a estudiar los fenmenos fsicos,
desde nuevos puntos de vista.
El nombre de la nueva ciencia fu sugerido por
E. C. Bingham en un congreso sobre plsticos cele-
brado en 1928 en Easton, Pensilvania, y defini la reo-
loga como una ciencia dedicada al estudio de la
deformacin y del flujo de la materia. En dicho con-
greso se cre la Sociedad Americana de Reologa,
pero el impulso definitivo fui debido a que las nece-
sidades guerreras plantearon nuevos y urgentes pro-
blemas relacionados con la reologa, lo cual indujo a
la creacin en Inglaterra, el ao 1940, del Club de
Reologistas Britnicos bajo la presidencia del pro-
fessor Sir Geoffrey Taylor y de los doctores Harrison
Lang y Scott Blair, y que cuenta ya con unos 300
miembros.
Fenmenos de tal importancia y tan mal conocidos
como las contracciones del cemento o el comporta-
miento del caucho sometido a tensiones, han de ser
estudiados por los mtodos reolgicos. Como ejemplo
de los hechos que caen dentro del dominio de la reo-
logia citaremos, adems, uno que se ha estudiado re-
cientemente, y se conoce con el nombre de elasticidad
de flujo (flow elasticity). Es un fenmeno que se
observa muy fcilmente, pero cuya explicacin no es
sencilla. Tmese una disolucin coloidal consistente

Vol. I, Fasc. 8 GAZETA DE F S I CA Julho, 1948
destinado ao estudo da medida das intensidades,
utilizando redes de difrao, interfermetros e micro-
fotmetros, sendo como nos captulos anteriores a ex-
posio clara e perfeitamente acessvel. O autor
conseguiu assim dar num livro relativamente pequeno
(cerca de 300 pginas), as bases essenciais para espec-
troscopia de alta preciso. L. S.
Ol i ver Heavi si de and the methemati cal theory
of electrical communications.
por Sir George Lee
Pub. por The British Council.
Esta publicao faz parte duma srie dedicada a
honrar a memria de ilustres homens de cincia brit-
nicos. uma iniciativa que por todos os motivos
merece sinceros aplausos e por isso mesmo de lamen-
tar que restries desconhecidas imponham a essas
obras uma extenso demasiadamente reduzida.
o caso deste livrinho de 30 pginas dedicado a
Heaviside.
Deste modo, apesar do seu valor pessoal, o autor
no consegue tornar o seu trabalho atraente para quem
no tenha priori um interesse determinado pelo
homenageado. E nem se fica sabendo qual foi preci-
samente a obra de Heaviside nem to pouco se in-
formado da intensidade das polmicas provocadas,
nomeadamente pelo seu clculo operational, e que
poderiam ser motivo de considerao de alto valor
formativo.
Em nossa opinio, o autor desperdia o pouco es-
pao de que dispe alongando-se sobre honrarias e
prmios cujo significado (e era s este que importava,
at na prpria opinio de Heaviside) poderia ter sido
condensado numa ou duas pginas.
No entanto o autor fez sem dvida um srio esforo
de documentao reproduzindo numerosos e interes-
santes trechos de cartas e de artigos, cuja leitura
proveitosa. A. G.
NOTICIRIO
Prof. Dr. A. Van Itterbeek
Este nosso colaborador, fsico de grande valor e
professor da Universidade catlica de Louvain deu-
nos ltimamente a alegria de vir visitar nosso pas.
Mas o que mais nos surpreendeu foi tal viagem ter
sido subvencionada por uma firma industrial, uma
entidade particular.
O convite partiu da Sociedade Corticeira Robinson
Bros. Lda, de Portalegre. A importncia actual da
Fsica para o progresso tal, que at uma fbrica de
cortia, at em Portugal, considera til a visita dum
fsico!
256
Ns, s podemos apoiar um tal ponto de vista como,
tambm, devemos lamentar que nos ltimos anos, em
que a Fsica se tornou o que todos sabem, um nico
fsico nos tenha visitado, e esse mesmo em condies
entre ns naturalmente to imprevistas.
Graas ainda a uma viso que nunca ser de mais
enaltecer, os administradores daquela Sociedade Cor-
ticeira pretenderam proporcionar aos estudiosos por-
tugueses a oportunidade de ouvirem o mestre estran-
geiro e promoveram com esse fim, fim puramente
cultural, 3 conferncias do Prof. Van Itterbeek, duas
em Lisboa e uma no Porto, sobre assuntos da sua es-
pecialidade, isto , ultra-sons e baixas temperaturas.
Foi curiosa a vinda deste professor belga!
Convidado inicialmente por uma Sociedade Corti-
ceira, depressa recebeu o alto patrocnio da Legao
da Blgica em Lisboa assim como do Instituto para
a Alta Cultura (este ofereceu-lhe mesmo um almoo
oficial de despedida), foi convidado pela Universidade
do Porto a fazer uma conferncia na sua Faculdade
de Cincias e recebeu nesta inequvocas provas de
considerao de numerosos professores da mesma, fez
ainda a convite, duas conferncias no Instituto Supe-
rior Tcnico de Lisboa, etc.
Porque no falou, nem foi recebido, na Faculdade
de Cincias de Lisboa? No sabemos ao certo ( to
estranho) mas ouvimos dizer que a Faculdade no
dispunha de salas e que, alm disso, as conferncias
s serviam para perturbar os alunos em vsperas de
exames.
possvel que esta explicao no passe duma
blague mas, seja como for, de lamentar que a
nossa Faculdade de Cincias de Lisboa tenha votado
ao ostracismo um sbio da categoria do Prof. A. Van
Itterbeek, categoria indiscutvel e bem conhecida.
De facto, pouco depois do seu regresso Blgica,
aquele Professor era oficialmente convidado para
Director do primeiro Laboratrio de Frio do mundo,
o Instituto Kamerlingh Omnes, de Leiden (Holanda),
cujos anteriores directores foram Omnes, Keesom e
De Haas. A. G.
Assinaturas
Em virtude das dificuldades financeiras em que a
Gazeta de Fsica se encontra, e para no termos que
recorrer a subsdios de apoio, avisamos os nossos pre-
zados assinantes de que o preo da assinatura (4 n-
meros 1 ano), a partir do prximo n. 9, passar a
ser de 40$00.
Nota
Devido dificuldade de obter papel semelhante ao
usado nos nmeros anteriores, fomos forados a atra-
zar mais a saida da revista, para no termos que
recorrer a um papel um pouco inferior. Deste atrazo
pedimos desculpa aos nossos assinantes.
Dem o vosso apoi o i nvest i gao ci ent i f i ca
Anunci ar na Gazet a de F si ca cont ri bui r para a sua prosperi dade

You might also like